Download as pdf or txt
Download as pdf or txt
You are on page 1of 58

Prepp IAS Upcoming courses for

UPSC CSE Mains 2023

Course Link:
https://iasexam.prepp.in/products/upsc-2023-mains-mentorship-pro
gram/
Course Link:
https://iasexam.prepp.in/products/upsc-2023-mains-test-series/
1

Most Probable Topics for UPSC CSE 2023 - History


1. Ajanta caves
2. Foreign travellers
3. Sangam Literature
4. Qutb Shahi architecture
5. Maratha Military
6. Lost wax technique
7. Avalokiteshwara Bodhisattva
8. Banawali
9. Keeladi excavation
10. 1857 Revolt
11. Temple entry movement
12. Charter Acts
13. Non-Cooperation Movement
14. Gentoo code
15. Raja Rammohan Roy
16. Lord Metcalfe
17. Sangam Terminology
18. Hindustani music
19. Ashokan edicts
20. Folk dances of India
21. Folk Theatre of India
22. Maratha Confederacy
23. NarasimhaVarman-I
24. Dravida style of temple architecture
25. Buddhism Mudras
26. Jagannath Temple
27. Fatehpur Sikri
28. Sankalp Bhumi Banyan tree campus
29. Battle of Saraighat
30. Commissions for Education
31. Quit India Movement
32. Provisional government of Free India
33. Gandhiji in South Africa
34. Dhondo Keshav Karve
35. Sessions of Indian National Congress (INC)
36. Swaraj party
37. August Offer
38. Official Secret Act, 1923
39. Ambedkar circuit
40. Savitri Bai Phule
41. Gandhiji - satyagraha
42. Hindu-German conspiracy
43. Mangarh massacre
44. Cornwallis code, 1793
45. Khilafat Movement
46. Trade organizations
47. Munda Rebellion
48. Alipore conspiracy case
49. Mulshi Peta Satyagraha
50. Important books/journals
51. Swami Dayanand Saraswati
52. Home Rule Leagues
53. Communal Award
54. Regulating act of 1773
55. Government of India Act, 1919
2

Q.1) With reference to the Ajanta caves, which one of the following statements is not correct?

a) Captain John Smith is credited with being the first European to discover the Ajanta Caves.
b) Majority of the caves of Ajanta belong to Hinduism and only a few caves belong to Buddhism.
c) The earliest caves of Ajanta were built during the reign of Satavahana kings.
d) Some of the paintings in Ajanta were made during the Gupta period.

Answer: (b)

Ajanta Caves:
● Captain Smith of the British Army was on a private tiger hunting expedition in the summer of 1819
when he discovered the Ajanta caves. Captain John Smith is credited with being the first European
to discover the Ajanta Caves. So, option (a) is correct.
● Ajanta Caves exemplifies one of the greatest achievements in
ancient Buddhist rock-cut architecture. The artistic traditions at
Ajanta present an important and rare specimen of art, architecture,
painting, and socio-cultural, religious and political history of
contemporary society in India. The development of Buddhism
manifested through the architecture, sculptures, and paintings
is unique and bears testimony to the importance of Ajanta as a
major hub of such activities. So, option (b) is not correct.
● The caves at Ajanta are excavated out of a vertical cliff above the
left bank of the river Waghora in the hills of Ajanta. They are thirty
in number, including the unfinished ones, of which five (caves 9,
10, 19, 26 and 29) are chaitya grihas (sanctuary) and the rest,
sangharamas or viharas (monastery).

● The caves are numbered in order to identify them but that does
not mean that they are constructed in chronological order.
● Caves constructed during Satavahana Period:
○ The caves 9, 10, 12, 13, and 15A were constructed
during this period and are considered as the earliest
caves. So, option (c) is correct.
○ These caves belong to the Hinayana sect of Buddhism.
○ Out of these caves, cave 9 and cave 10 have Buddhist Stupas which have worship halls and
caves 12, 13, and 15A have viharas.

● Caves constructed during Vataka Period:


○ According to historians and various studies, it
has been found that the second phase of the
construction of Ajanta caves started during the reign
of Harisena, a king of Vataka dynasty.
○ The caves constructed during this period belonged
to the Mahayana sect of Buddhism.
○ The caves that were constructed during this period
are 1 to 8, 11, and 14 to 29. Out of these caves, 19,
26, and 29 are chaitya grihas and rest are viharas.
○ After the death of Harisena, construction of the
caves came to a halt.
● Paintings:
○ There are several paintings that can be found in
the Ajanta caves.
○ People can mostly find Mural paintings that were
made during both the phases.
○ Ancient paintings show that they were made during the Satvahana period. Some of the
caves have paintings from the Gupta period and later. So, option (d) is correct.
Therefore, option (b) is the correct answer.
3

Q.2) Consider the following pairs regarding travellers who visited India during the eras of different rulers:
Pair Traveler Ruler

1. Abdur Razzaq Devaraya II

2. Nicolo Conti Chandragupta Maurya

3. Domingo Paes Aurangzeb

4. Thomas Roe Akbar

How many pairs given above is/are correctly matched?

a) Only one pair


b) Only two pairs
c) Only three pairs
d) Al the four above

Answer: (a)

● Nicolo Conti (1420 A.D.– 1421 A.D.)


○ Italian explorer and merchant Niccolo Conti travelled to India in the year 1420, perhaps after
Deva Raya II assumed the throne of the Vijayanagara state. So, pair 2 is not correctly
matched.
○ Niccolo visited Sonargaon and Chittagong (in modern-day Bangladesh), then travelled by
land to Arakan (now Rakhine State, Burma).
○ The southernmost point that Conti ventured to was Ceylon. He made several stops,
notably in Cochin and Calicut, along the Malabar Coast of India (Kozhikode).
○ Before travelling to the southern coast of the Arabian Peninsula and the city of Aden, he
returned to Cambay.
● Abdur Razzaq (1443-1444 A.D.)
○ An Islamic scholar and philosopher named Abdur Razzak paid a visit to India while Deva
Raya II, the most illustrious ruler of the Sangama dynasty, was in power. So, pair 1 is
correctly matched.
○ The people of Calicut, whom he described as having bad hygiene and practicing
polyandry, did not impress him.
○ Since the Vijayanagar King had called him to his dominion, his time in Calicut was brief.
○ Razzak passed through Mangalore before arriving in Vijayanagara.
○ His book entitled 'Matla-us-Sadainwa Majma-ul-Bahrain' described the life and events in
Calicut under the Zamorin and also of the Ancient City of Vijayanagar at Hampi.
● Domingo Paes (1520-1522 A.D.)
○ He was a Portuguese trader, writer, and explorer who visited India between 1520 and 1522
AD and provided the most complete account of the ancient city of Hampi under the reign of
King Krishnadeva Raya of the Tuluva Dynasty of the Vijayanagara Empire. So, pair 3 is
not correctly matched.
○ In his book "Chronica dos reis de Bisnaga," he chronicled his voyage and gave in-depth
information on the Vijayanagara Empire.
● Thomas Roe( 1615 A.D. – 1619 A.D)
○ Thomas Roe served as a diplomat.
○ He visited India in 1615, during Jahangir’s rule. He went to Surat to look for security for an
English company. So, pair 4 is not correctly matched.
○ He left behind a priceless contribution to Indian history with his “Journal of the Mission to the
Mughal Empire.”
○ He came to seek protection for an English factory at Surat.
Therefore, option (a) is the correct answer.
4

Q.3) It was one of the earliest epic poems in Tamil, written in the 5th–6th century AD. It tells the story of young
merchant Kovalan, his marriage to the virtuous Kannagi and his love for the courtesan Matavi. Which of the
following texts has been described in the above passage?

a) Amuktamalyada
b) Ratnavali
c) Kundalakesi
d) Silappathikaram

Answer: (d)

● Silappathikaram, also spelled Silappatikaram is the earliest epic poem in Tamil, written in the
5th–6th century AD by Ilango Adigal.
○ The Silappathikaram tells the story of the young merchant Kovalan and his wife Kannagi. It
tells about Kovalan’s love for the courtesan Matavi, and his consequent ruin and exile in
Mathurai.
○ In Mathurai he is unjustly executed after trying to sell his wife’s anklet to a wicked goldsmith
who had stolen the queen’s anklet and charged Kovalan with theft.
○ Kannaki comes to Maturai, proves Kovalan’s innocence, then tears off one breast and throws it
at the kingdom of Maturai, which goes up in flames. Such is the power of a faithful wife. So,
option (d) is correct.
● Amuktamalyada is an epic poem in Telugu composed by Krishnadevaraya of the Vijayanagar
Dynasty. So, option (a) is not correct.
● Harsha wrote three Sanskrit plays- Nagananda, Ratnavali, and Priyadarshika. So, option (b) is not
correct.
● Kundalakesi, also known as Kuntalakeciviruttam, is a Tamil epic authored by Nathakuthanaar about
the 10th century. So, option (c) is not correct.
Sangam Period:
● The period roughly between the 3rd century B.C. and 3rd century A.D. in South India (the area lying
to the south of river Krishna and Tungabhadra) is known as Sangam Period.
● It has been named after the Sangam academies held during that period that flourished under the royal
patronage of the Pandya kings of Madurai.
● At the sangams, eminent scholars assembled and functioned as the board of censors and the choicest
literature was rendered in the nature of anthologies.
● These literary works were the earliest specimens of Dravidian literature.
● South India, during the Sangam Age, was ruled by three dynasties-the Cheras, Cholas and
Pandyas.
Therefore, option (d) is the correct answer.

Q.4) Which of the following monuments is/are not belong to Qutb Shahi architecture?

a) Golconda forts.
b) Charminar
c) Qutb shahi tombs.
d) Bijapur Fort.

Answer: (d)

Qutb Shahi Architecture:


● The city of Hyderabad served as the capital of the Qutb Shahis, the Asaf Jahi Nizams and is now
the capital of the state of Telangana.
● Qutb Shahi Islamic Sultanate was one of the five prominent dynasties that emerged in the Deccan
following the downfall of the Bahmani Dynasty in 1518 A.D.
● Seven rulers of the Dynasty ruled for 170 years and successfully resisted the Mughal attack till 1687
A.D.
● It was the last kingdom to be absorbed in the expanding Mughal Empire.
● The monuments of the Qutb Shahi period represent different building typologies.
5

● Golconda Fort, Qutb Shahi Tombs and Charminar, located in Hyderabad, are the landmarks that
together symbolize the Qutb Shahi Dynasty (1518 A.D. to 1687 A.D.). So, options a, b and c are
correct.

Bijapur Fort:
● Bijapur fort is a very big fort with many structures inside.
● The fort was built in 1565 by the founder of the Adil Shahi dynasty. So, option d is not correct.
● There are five main gates of the fort and each has ten bastions.
● A moat surrounds the fort which was made so that the enemy could not enter the fort. The moat has a
width of 50 feet.

Therefore, option (d) is the correct answer.


6

Q.5) With reference to Maratha military, Consider the following pairs:


S. No. Terminology Meaning

1. Ghuraw Rowing boat of 40-50 rovers

2. Galliwat Boats laden with Guns

3. Jumladar Head of five nayaks

4. Sar naubat Incharge of the army.


How many pairs given above is/are correctly matched?

a) One pair only


b) Two pairs only
c) Three pairs only
d) All the four pairs

Answer: (b)

Military administration of Marathas:


● Shivaji organised a disciplined and efficient army.
● The ordinary soldiers were paid in cash, but big chief and military commander were paid through
jagir grants (Saranjam or Mokasa).
● The army consists of:
○ Infantry i.e. Mavali foot soldiers;
○ Cavalry i.e. Horse riders & equipment holders;
○ Navy.
● The army was an effective instrument of policies of Marathas State where rapidity of movement was the
most important factor.
○ Only in the rainy season, the army gets rested, otherwise the rest of the year was engaged in
expeditions.
● Pindaries were allowed to accompany the army who were allowed to collect “Pal-Patti” which was
25% of war booty.
● Military Personnel of Maratha army:
○ Sar-i-Naubat (Senapati)- Incharge of the army. So, pair 4 is correctly matched.
○ Qiladars- Officers of Forts
○ Nayak- Head of the member unit of infantry
○ Jumladar- Head of five Nayaks. So, pair 3 is correctly matched.
○ Ghuraw- Boats laden with guns. So, pair 1 is not correctly matched.
○ Gallivat- Rowing boats 40-50 rowers. So, pair 2 is not correctly matched.
○ Paik- Foot Soldiers
Therefore, option (b) is the correct answer.

Q.6) Bronze casting was a widespread practice during the Indus Valley Civilization, particularly at Harappa.
Bronze statues were made by using which of the following techniques?

a) Lost wax technique


b) Slip Casting technique
c) Hand building technique
d) Stone carving

Answer: (a)
Lost Wax Technique:
● Bronze casting was a widespread practice during the Indus Valley Civilization, particularly at
Harappa.
○ Bronze statues were made by the “lost wax technique”. So, option (a) is correct.
7

● This practice is still prevalent in many parts of the country particularly Himachal Pradesh, Odisha,
Bihar, Madhya Pradesh and West Bengal.
● Under this technique, the bee wax is first melted over an open fire and then strained through a fine
cloth into cold water.
● The bee wax immediately solidifies and then it is passed through a pharni, so that the wax comes
out of it in the shape of noodle-like wires.
● These wax wires are now used to make a shape of the entire image first.
● After that, this image is covered with a paste of clay, sand and other materials such as cow dung.
● On one side, an opening is kept. When it becomes dry, the wax is heated and the molten wax is drained
out through a tiny hole.
● The hollow mould thus created is filled with molten metal which takes the original shape of the
object.
Slip Casting technique:
● Slip casting is the forming of ceramics by pouring or pumping deflocculated (water reduced) clay
slurry into plaster molds. So, option (b) is not correct.
● In the process, the absorbent plaster pulls water from the slurry, and over a period (e.g. 20 minutes) a
layer builds up against the mold surface.
● The slurry is then poured out and within a short time, the item shrinks slightly and can be removed from
the mold.
● This process is flexible, capable of producing fine delicate porcelain items yet heavy utilitarian
objects can also be done.
● The casting process is the best where very thin-walled or complex-shaped pieces are needed.
Hand building techniques:
● Handbuilding is an ancient pottery-making technique that involves creating forms without a
pottery wheel, using the hands, fingers, and simple tools.
● The most common handbuilding techniques are pinch pottery, coil building, and slab building. So,
option (c) is not correct.
Stone Carving
● This form of sculpting is common in architectural pieces as well as the making of figures.
● The type of stone used depends on the region. This method of sculpting became prominent in the
post mauryan phase. So, option (d) is not correct.
Therefore, option (a) is the correct answer.

Q.7) With reference to Avalokiteshwara Bodhisattva, consider the following statements:


1. Avalokiteshvara represents infinite compassion and mercy.
2. Avalokiteshvara’s right hand is in the gesture of gift granting, and in his left hand he holds a
long-stemmed lotus.
3. He represents the idea of welfare as he delayed his own transformation into buddha.
Which of the statements given above is/are correct?

a) 1 and 2 only
b) 2 only
c) 1 and 3 only
d) 1, 2 and 3

Answer: (d)
Explanation:

● The ideal of Bodhisattva beholds the highest moral principle of intellect and discipline in
Buddhism for the sake of happiness of all the other beings.
● Avalokiteshvara means one who can see all.
● He represents infinite compassion and mercy. So, statement 1 is correct.
● Avalokiteshvara is shown here sitting on a lion throne and arrayed in jewels like a prince. Above
his head is a parasol, an ancient symbol of royal status. Beside his head are lotus-borne stupas,
topped with sun and crescent moons. Avalokiteshvara’s right hand is in the gesture of gift
granting, and in his left hand he holds a long-stemmed lotus. So, statement 2 is correct.
8

● In his elaborate headdress is an image of the Buddha Amitabha. (Amitabha presides over the
western Pure Land, a kind of Buddhist paradise. The worship of Amitabha became very popular in
East Asian Buddhism).
● He is the most popular Bodhisattva of all.
● He represents the ideal of welfare in which he
postpones his own transformation into Buddha
to help others. So, statement 3 is correct.
There are 8 important Bodhisattvas who represent
different aspects of Buddha. They are as following:

1. Avalokitesvara (Buddha’s Compassion): He is


the most popular Bodhisattva of all. He
represents the ideal of welfare in which he
postpones his own transformation into Buddha to
help others.
2. Manjusri (Buddha’s Wisdom): The literal meaning
of his name is ‘Gentle Glory’. He represents
Pragya or transcendental wisdom at a very early
age.
3. Samantabhadra (Buddha’s Aspirations): The
meaning of Samantabhadra is ‘Universal Worthy’
and he is associated with practice and
meditation. He made ten vows to fulfil his journey
of becoming a Buddha.
4. Ksitigarbha (Buddha’s Merit): He is popular as a
Buddhist monk in the mostly East Asia. While the
name literally means Earth Womb, he is revered
as the guardian of children and patron deity of
deceased children. He carries a staff.
5. Maitreya (Buddha’s Activity): He is also known as
Ajita Boddhisattva. It is believed that he will arrive
when people will forget the path of righteousness. He is considered as the successor of Gautam
Buddha. In Buddhist traditions, he is revered as someone who will achieve complete enlightenment and
will teach dharma to people.
6. Vajrapani (Buddha’s Power): He is described as one of the 3 protective deities around Buddha, others
are Manjusri and Avlokiteshwara. Vajrapani represents Buddha’s power while Buddha’s wisdom is
symbolised by Manjushri, and Avalokiteshwara manifests as Buddha’s compassion.
7. Sarvanivarana-Vishkambhin (Buddha’s Qualities): He is invoked to remove or eliminate all the
obstacles to insure a successful meditation.
8. Akasagarbha (Buddha’s Blessings): He is boundless as space. He was the twin brother of
Ksitigarbha. He manifests as a blessing.

Therefore, option (d) is the correct answer.

Q.8) This archaeological site was discovered by R.S.Bisht in 1974. It was established on the left bank of the dry
river Saraswati. Several dwellings here include traces of fire altars. Evidence of both pre-Harappan and
Harappan culture have been found here. A Replica of a plough and a clay figure of Mother Goddess was found
here. The above mentioned passage refers to which of the following sites of the Harappan civilisations?

a) Kalibangan
b) Banawali
c) Rakhigarhi
d) Chanhudaro

Answer: (b)
9

Banawali

● Banawali is an important Indus Valley


Civilization archaeological site discovered by
R.S.Bisht in 1974.
● It is situated in Haryana's Fatehabad district.
It is roughly 120 kilometers northeast of
Kalibangan.
● The city is a well-built fort town with a
4.5-meter-high defense wall. The dwellings
were built using burnt and moulded brick kilns.
● The pottery unearthed from the Indus Valley
Civilization site at Banawali is extremely
comparable to that found in Kalibangan.
● Banawali, formerly known as Vanavali, is a
hamlet and archaeological site located around
15 kilometers from Fatehabad district in
Haryana, India.
● Several seals and weights were discovered
in a multi-roomed residence with a kitchen and
lavatory, indicating that the property's owner was presumably a merchant.
● A larger home revealed a great quantity of gold beads, lapis lazuli, carnelian, small weights, and a
'touch stone'-like stone with gold streaks, indicating that the house belonged to a jeweler or ornament
manufacturer.
● The Replica of a plough was found in the Banawali district. The clay figure of the mother goddess
was also found there.
● Several dwellings in Banawali include traces of fire altars, which were also coupled with apsidal
constructions indicating ceremonial reasons.
● Evidence of both pre-Harappan and Harappan culture are found here. So, option (b) is correct.
Kalibangan:
● The archaeological site of Kalibangan in Rajasthan has discovered evidence of the world's oldest
plowed agricultural field.
● It was discovered outside the fort, southeast of the pre-Harappan village.
● All residences opened out to at least two or three roads or lanes due to the grid pattern of the town
layout, which resembled a chessboard.
● A handful of seals from this period have been discovered. A cylindrical seal featuring a female figure
between two masculine characters battling or menacing with spears is the most notable
Rakhigarhi:
● Rakhigarhi is the largest Harappan site in the Indian subcontinent.
● At Rakhigarhi, the excavations are being done to trace its beginnings and to study its gradual evolution
from 6000 BCE (Pre-Harappan phase) to 2500 BCE.
● The site was excavated by Amarendra Nath of ASI
● The archaeological excavations revealed the mature Harappan phase represented by planned
township having mud-brick as well as burnt-brick houses with a proper drainage system.
● The ceramic industry was represented by red ware, which included dish-on-stand, vase, perforated
jar among others.
● Animal sacrificial pit lined with mud-brick and triangular and circular fire altars on the mud floor
have also been excavated that signifies the ritual system of Harappans.
Chanhudaro:
● At Chanhudaro, evidence of shell works was discovered, and bangles and ladles were created there.
● Harappan seals were manufactured in larger towns such as Harappa, Mohenjo Daro, and Chanhudaro
that were part of the administrative network.
● The Chanhu Daro has provided evidence of factories producing a variety of figurines, seals, toys,
and bone instruments, suggesting that it was a village with a large number of craftsmen and an
industrial town.
Therefore, option (b) is the correct answer.
10

Q.9) With reference to Keeladi excavation, consider the following statements:


1. These excavations show urban life and settlements in Tamil Nadu during the Early Historic Period.
2. Keeladi was a center of the dyeing industry and a glass bead industry.
3. The site is located on the banks of the Krishna River.
Which of the statements given above is/are correct?

a) 3 only
b) 2 only
c) 1 and 2 only
d) 1, 2, and 3

Answer: (c)
Explanation:

● Keeladi is a tiny hamlet in the Sivaganga district in south Tamil Nadu.


● It is about 12 km south-east to the temple city of Madurai.
● The Sangam age is a period of history in ancient Tamil Nadu which was believed to be from the third
century BCE to the third century CE.
● The name is derived from the renowned Sangam poets of Madurai from that time.
● It comes across as an industrious and advanced civilisation and has given evidence of urban life
and settlements in Tamil Nadu during the Early Historic Period. So, statement 1 is correct.
● There also existed a dyeing industry and a glass
bead industry. Gold ornaments, copper articles,
semi-precious stones, shell bangles, ivory bangles
and ivory combs reflect the artistic, culturally rich and
prosperous lifestyle of the Keeladi people. So,
statement 2 is correct.
● The excavations from here prove that an urban
civilisation existed in Tamil Nadu in the Sangam age
on the banks of the Vaigai River. So, statement 3 is
not correct.
Key facts about the Sangam period

● The word ‘Sangam’ is the Tamil form of the


Sanskrit word Sangha which means a group of
persons or an association.
● This sangama was an academy of poets who
flourished in three different periods and different
places under the patronage of the Pandyan kings.
● The Sangam literature, which was largely
consolidated from the third Sangam, sheds light
on people's living conditions at the start of the
Christian era.
● It gives information about the secular matter
relating to public and social activities like government, war charity, trade, agriculture, etc.
● Sangam literature consists of the earliest Tamil works (Tolkappiyam), the ten poems (Pattupattu),
the eight anthologies (Ettutogai) and the eighteen minor works (Padinenkilkanakku).

Therefore, option (c) is the correct answer.


11

Q.10) He was one of the most notable Indian freedom fighters and a general in the Rebellion of 1857. In May
1857, he won the battle over the Indian troops of the East India Company at Kanpur. He forced General
Windham to retreat from the city of Gwalior. He collaborated with Rani Lakshmi Bai of Jhansi to seize Gwalior. He
was defeated by Sir Colin Campbell on December 6, 1857. Who among the following is referred to in the above
passage?

a) Nana Saheb
b) Tantia Tope
c) Kunwar Singh
d) Azimullah Khan

Answer: (b)

Nana Saheb:
● He was born in Bithoor of Kanpur District in present day Uttar Pradesh in May 1824.
● He was initially named as Nana Govinda Dhondu Pant.
● His father travelled to the court of the Peshwa Baji Rao II in Pune and became his court official.
● Tatya Tope and Manikarnika Tambe (later Rani Laxmibai of Jhansi) were his childhood friends.
● The Doctrine of Lapse established by Lord Dalhousie under which any Indian State under the
control of the British or any vassal of the British without its ruler having an heir would be annexed by the
British.
● Hence after the death of Peshwa, the British stopped giving pension to his adopted son Nana
Saheb and refused to accept him as the heir (since he was adopted).
● Britishers refused the claim of Nana Saheb to be the next peshwa, despite being stated as the heir in
the will of Baji Rao II.
● This incident led Nana Saheb to take part in the revolt of 1857. So, option (a) is not correct.
Tatya Tope:
● Also known as Ramachandra Pandurang Tope, he was one of the most notable Indian freedom
fighters and a general in the Rebellion of 1857.
● Tatya Tope was an intimate friend and the right hand of Nana Sahib, the adopted son of Peshwa
● In May 1857, Tatya Tope won the battle over the Indian troops of the East India Company at
Kanpur
● He forced General Windham to retreat from the city of Gwalior.
● He collaborated with Rani Lakshmi Bai of Jhansi to seize Gwalior.
● Tatya Tope was defeated by Sir Colin Campbell (later Baron Clyde) on December 6, 1857. He was
hanged on April 18, 1859, in General Meade’s camp at Shivpuri. So, option (b) is correct.
Kunwar Singh:
● He belonged to a family of the Ujjainiya clan of the Parmar Rajputs of Jagdispur, currently a part of
Bhojpur district, Bihar.
● He was the chief organiser of the fight against the British in Bihar. He is popularly known as Veer
Kunwar Singh.
● He was assisted by both his brother, Babu Amar Singh and his commander-in-chief, Hare Krishna
Singh.
● He gave a good fight and harried British forces for nearly a year and remained invincible until the
end. He was an expert in the art of guerilla warfare. So, option (c) is not correct.
● He passed away on 26th April 1858.
Azimullah khan:
● Azimullah played a significant role during the Great Rise of 1857.
● His powerful poetic expressions brought about an atmosphere of closely integrated coexistence of
Hindu and Muslim communities. At the same time he held the reins of the revolution in his hands and
controlled most of the events himself.
● The patriotic song was composed by revolutionary Azimullah Khan and published in a news daily
“ Payme Azadi” patronized by Nana Sahib. One original copy of it has been preserved by the British
Museum, London. So, option (d) is not correct.
Therefore, option (b) is the correct answer.
12

Q.11) With regard to temple entry movement in pre-independence India, consider the following pairs:
S. No. Temple Entry Movement Led by

1. Vaikom Satyagraha E V Ramaswamy Naicker

2. Aruvippuram movement Shri Narayan Guru

3. Kalaram Temple entry movement Dr. B. R. Ambedkar

4. Satyagraha in Guruvayur K. Kelappan


How many pairs given above is/are correctly matched?

a) One pair only


b) Two Pairs only
c) Three Pairs only
d) All the four pairs

Answer: (d)

Vaikom Satyagraha:
● The movement began on 30th March 1924.
● At the Vaikom Mahadeva Temple, there was a board that denied the entry of “lower caste” people
(avarnas).
● The Satyagrahis made batches of three and entered the temple. They were resisted and arrested by the
police.
● The movement gained prominence in the whole of India and support came from far and wide.
● After discussions with caste Hindu people failed, the leaders again started the movement. Leaders T K
Madhavan and K P Kesava Menon were arrested.
● E V Ramaswami Naicker (Periyar) came from Tamil Nadu to support the movement and then he
was arrested.
● On 1st October 1924, a group of savarnas (forward castes) marched in a procession and submitted a
petition to the Regent Maharani Sethu Lakshmi Bai of Travancore with approximately 25000 signatures
for allowing entry to the temple for everyone.
Events during Satyagraha:
● Start of Satyagraha: Satyagrahis walked in procession towards the forbidden public roads.
● Role of George Joseph: He briefly led the agitation. He also wrote to ‘Periyar’ E.V. Ramasamy, then
Tamil Nadu Pradesh Congress Committee president, led the campaign.
● Arrival of Mahatma Gandhi: at Vaikom in 1925. He also met Queen Sethulakshmi Bai (Maharani
Regent) of Travancore.
○ Their meeting resulted in a royal proclamation by which all the public roads to Vaikom
Mahadeva Temple were opened to all castes.
● Movement under Periyar: Under the guidance of Mahatma Gandhi and C. Rajagopalachari, Periyar
then led the movement till its conclusion.
○ He mobilized volunteers and garnered public support.
○ Part of an eight-member delegation constituted to meet the Diwan of Travancore.
○ He held meetings with Mahatma Gandhi, Sree Narayana Guru, Swami Shraddhananda, and C.
Rajagopalachari.
Outcome of the movement:
● Compromised solution: Vaikom Satyagraha was officially withdrawn after consultations between
Gandhi and W.H. Pitt, the then police commissioner of Travancore.
○ A compromise was reached following the release of all prisoners and grant of access to three
of the four roads to the Vaikom temple.
● Rift between Mahatma Gandhi and Periyar: While Gandhiji saw it as Hindu reformist movement,
Periyar called it a fight against caste-based atrocities (also quit congress months later).
● Temple Entry Proclamation (1936): was signed by Maharaja of Travancore which removed the age-old
ban on the entry of marginalised castes into the temples of Travancore.
Shree Narayan Guru Dharma Paripalana Movement:
13

● The Ezhavas were Kerala's most populous caste, accounting for 26% of the total population.
● On Sivaratri in 1888, Narayana Guru, himself of the Ezhava caste, took a stone from the Neyyar river
and installed it as a Sivalinga at Aruvippuram.
● Narayan Guru himself created a Shivalinga at Aruvippuram at Shivaratri in 1888, using a stone from
the Neyyar river. So, pair 2 is correctly matched.
● He triggered a revolution in Kerala that resulted in the removal of many sorts of bigotry as a result of
this.
● "One Caste, One Religion, One God for All," he coined the now-famous slogan. “Oru Jathi, Oru
Matham, Oru Daivan, Manushyanu''.
● He preached equality, but he did not feel that inequities should be used to carry out conversions and
thereby produce social upheaval.
● It was meant to demonstrate that the consecration of an idol was not limited to the upper castes.
● With this, he launched a revolution that resulted in the abolition of many discriminations in Kerala's
society.
● As a disciple of Narayana Guru, the movement (Aruvippuram Movement) drew the famous poet
Kumaran Asan.
● The Aruvippuram Kshetra Yogam was founded in 1889, with the intention of growing into a large
organization to assist the Ezhavas in their material and spiritual advancement.
● The Aruvippuram Sree Narayana Guru Dharma Paripalana Yogam was established in 1903 under
the Indian Companies Act, with Narayana Guru serving as its permanent chairman and Kumaran Asan
serving as its general secretary.

Objectives of SNDP Movement:


● To oppose the predominance of Brahmanas and other upper castes in general and demanded
equal rights for lower caste people.
○ To enhance the position of the backward castes in the caste hierarchy by copying
upper-caste lifestyles, such as higher education and distinguished occupations.
● 'Ethnic' awareness and politicization were generally established as a result of these activities among
diverse caste groupings.

Kalaram Temple Movement:

● Kalaram Temple entry movement formed a pivotal role in the Dalit movement in India. B. K
(Dadasaheb) Gaikwad and Dr. B. R. Ambedkar led a protest outside the temple on 2 March 1930,
in order to allow Dalits into the temple. So, pair 3 is correctly matched.
● The movement was to have a right to enter a temple, it was more towards having equal rights.
○ We don’t want to go to temples though but we should have rights.
● Kala Ram Temple is a beautiful temple in Old Nashik area, built by Sardar Rangarao Odhekar in
1788.
● Thousands of pilgrims visit the temple to take the blessings of Lord Ram. But until 1930 no dalit or
untouchable was allowed to enter the premises of the temple.
Satyagraha at Guruvayur:
● Guruvayur Satyagraha took place in 1931–32 and was a non-violent protest in the present
Thrissur district, now part of Kerala, which was an effort to allow entry for untouchables into the
Guruvayur Temple.
● It was led by K. Kelappan, who undertook a hunger strike for 12 days until it was abandoned
because of a request from Mahatma Gandhi and the Indian National Congress. So, pair 4 is
correctly matched.
Therefore, option (d) is the correct answer.
14

Q.12) With reference to Charter acts, consider the following statements:


1. The Charter act of 1813 empowered local bodies to impose taxes on persons.
2. The law member became a full time member of Governor General’s executive council with right to vote
under Charter Act of 1853.
3. The system of open competition for recruitment in civil services was introduced in the Charter act of
1833.
Which of the statements given above is/are correct?

a) 1 only
b) 2 and 3 only
c) 1 and 3 only
d) 1, 2 and 3

Answer: (d)

Charter Act of 1813:


Features:
● The Charter act of 1813 ended the monopoly of the East India Company in India, the company’s
monopoly in trade with China and trade in tea with India was kept intact.
● The company’s rule was extended to another 20 years.
● The act granted permission to the persons who wished to go to India for promoting moral and
religious improvements. (Christian Missionaries)
● This act regulated the company’s territorial revenues and commercial profits. It was asked to keep its
territorial and commercial accounts separate.
● The company’s dividend was fixed at 10.5% per annum.
● There was also a provision that the Company should invest Rs. 1 Lakh every year on the education
of Indians.
● It empowered the Local Governments in India to impose taxes on persons and to punish those who
did not pay them. So, statement 1 is correct.
Charter act of 1833:
Features:
● The charter act of 1833 legalized the British colonization of India.
○ It ended the activities of the East India Company as a commercial body, it became an
administrative body.
○ It provided that the company’s territories in India were held by the government ‘in trust for His
Majesty, His heirs and successors’.
● It made the Governor-General of Bengal as the Governor-General of India and vested in him all
civil and military powers.
○ This made Lord William Bentinck the first Governor-General of India.
● The Governors of Bombay and Madras lost their legislative powers. The Governor-General of India
had legislative powers over the entire British India.
● The laws made under the previous acts were called Regulations while laws made under this act
were called Acts.
● This Act introduced a system of open competition for selection of civil servants, and stated that
the Indians should not be debarred from holding any place, office and employment under the
Company. So, statement 3 is correct.
● It also provided that the Hailey bury college of London should make a quota to admit the future civil
servants. However, this system of open competition was not effectively operated in the near future.
● The Governor-General in council had the authority to amend, repeal or alter any law in British Indian
territories.
○ The Governor-General’s council was to have four members again; the fourth member had
limited powers only.
● For the first time, the Governor-General’s government was called Government of India and the
council was called India Council.
● Indian Law Commission was established to codify all Indian laws. The first Law Commission had Lord
Macaulay as its chairman.
● This act also directed the Governor General-in-Council to adopt measures to mitigate the state of
slavery, persisting in India since the sultanate Era.
15

Charter act of 1853:


Features:
● The Charter Act of 1853 empowered the British East India Company to retain the territories and
the revenues in India in trust for the crown not for any specified period, Unlike the previous charter
acts of 1793, 1813 and 1833 which renewed the charter for 20 years.
● This Act was passed when Lord Dalhousie was the Governor-General of India.
● Governor-General’s office :
○ It separated, for the first time, the legislative and executive functions of the Governor-
General’s council.
○ It provided for the addition of six new members called legislative councilors to the council
{12 in total}.
○ The Governor-General could nominate a vice president to the council and his assent is
required for all legislative actions.
○ It established a separate Governor-General’s legislative council which came to be known as
the Indian (Central) Legislative Council.
○ This legislative wing of the council functioned as a mini-Parliament, adopting the same
procedures as the British Parliament.
○ The Law member (fourth member) became a full member of the executive council with
the right to vote. So, statement 2 is correct.
○ The 12 members were – 1 Governor-General, 1 Commander-in-Chief, 4 members of the
Governor-General’s Council, 1 Chief Justice of the Supreme Court at Calcutta, 1 regular judge
of the Supreme Court at Calcutta, and 4 representative members drawn from company’s
servants with at least 10 years tenure, appointed by the local governments of Bengal, Bombay,
Madras and North Western Provinces.
○ It introduced an open competition system of selection and recruitment of civil servants. The
covenant civil service was thus thrown open to the Indians also. Accordingly, the Macaulay
Committee (the Committee on the Indian Civil Service) was appointed in 1854.
○ The number of Board of Directors was reduced from 24 to 18 out of which 6 people were
to be nominated by the British Crown.
○ This act served as the foundation of the modern parliamentary form of government. The
legislative wing of the Governor-General’s Council acted as a parliament on the model of the
British Parliament.
Therefore, option (d) is the correct answer.

Q.13) Which of the following events happened during the Non-Cooperation Movement?
1. Akali movement
2. Defiance of forest laws in Andhra
3. Panchayats were established for settling disputes
4. Boycott of British Titles and Goods
Select the correct answer using the codes given below:

a) 1 and 2 only
b) 2, 3 and 4 only
c) 3 and 4 only
d) 1, 2, 3 and 4

Answer: (d)

● The Non-Cooperation Movement was a significant phase of the Indian independence movement from
British rule.
● Various causes like the Home Rule Movement set the stage for the non-cooperation movement.
The Khilafat movement was another cause of this movement. Due to World War I, there was an
economic crisis and it was another reason behind this movement. The massacre of Jallianwala Bagh
and Rowlatt Act was also the reasons behind this movement. The tough rules of the British government
in India are behind this movement
Forms of protest:-
16

● Boycott of British Titles and Goods: The programme of non-cooperation consisted of a surrender of
British titles and honors, boycott of British Courts, Legislatures and educational institutions as well as
the boycott of foreign-made goods.
○ People lit public bonfires of foreign cloth. The imports of foreign cloth fell drastically
between 1920 and 1922. So, statement 4 is correct.
● Boycott of government affiliated schools and colleges
● Boycott of law courts
● Picketing of shops selling foreign cloth was also a major form of the
● Boycott could be extended to include resignation from government service and mass civil
disobedience including the non-payment of taxes.
● National schools and colleges were to be set up
● Panchayats were to be established for settling disputes. So, statement 3 is correct.
● Hand-spinning and weaving was to be encouraged
● Kisan sabhas: In the Avadh area of U.P., where kisan sabhas and a kisan movement had been
gathering strength since 1918 and with Non-cooperation movement it became difficult to distinguish
between a Non cooperation meeting and a kisan meeting.
● In Malabar in Kerala, Non cooperation and Khilafat propaganda helped to arouse the Muslims tenants
against their landlords.
● About Moplah Rebellion:
○ The Khilafat meetings in Malabar incited communal feelings among the Moplahs and it
became a movement directed against the British as well as the Hindu landlords of Malabar.
○ There was large-scale violence that saw systematic persecution of Hindus and British
officials. Many homes and temples were destroyed.
○ The prominent leaders of the rebellion were Ali Musaliyar and Variyankunnath
Kunjahammed Haji.
● Charkhas were popularized on a wide scale and khadi became the uniform of the national movement.
● Defiance of forest laws became popular in Andhra. So, statement 2 is correct.
● Peasants and tribals in some of the Rajasthan states began movements for securing better conditions of
life.
● Akali movement: In Punjab, the Akali Movement for taking control of the gurudwaras from the
corrupt mahants (priests) was a part of the general movement of Non-cooperation, and the Akalis
observed strict non-violence in the face of tremendous repression. So, statement 1 is correct.
Therefore, option (d) is the correct answer.

Q.14) With reference to the ‘Gentoo code’, consider the following statements:
1. It was a translation which was funded and encouraged by Cornwallis.
2. It was translated from Sanskrit to Persian and then in English.
Which of the statements given above is/are correct?

a) 1 only
b) 2 only
c) Both 1 and 2
d) Neither 1 nor 2

Answer: (b)

Gentoo Code:
● The Gentoo Code is a legal code translated from Sanskrit (in which it was known as
vivādārṇavasetu) into Persian by Brahmin scholars; and then from Persian into English by
Nathaniel Brassey Halhed, a British grammarian working for the East India Company. So, statement 2
is correct.
● The translation was funded and encouraged by Warren Hastings as a method of increasing
colonial hold over the Indies. So, statement 1 is not correct.
● It was printed privately by the East India Company in London in 1776 under the title A Code of
Gentoo Laws, or, Ordinations of the Pundits.
17

● Copies were not put on sale, but the Company did distribute them. In 1777 a pirate (and less luxurious
edition) was printed; and in 1781 a second edition appeared.
Therefore, option (b) is the correct answer.

Q.15) With reference to Raja Rammohan Roy, consider the following statements:
1. He set up the Atmiya Sabha in Calcutta to propagate the ideals of Vedanta.
2. Tuhfat-ul-Muwahhidin is a publication work of RamMohun Roy.
3. He wanted to establish a new religion as an alternative to Hinduism.
Select the correct answer using the code given below:

a) 1 only
b) 1 and 2 only
c) 2 and 3 only
d) 1, 2 and 3

Answer: (b)
Explanation:

Context: On Raja Ram Mohan Roy’s 250th birth anniversary, remembering the legacy of the father of Modern
Indian Renaissance.

Raja Ram Mohan Roy


● Born into a prosperous upper-caste Brahmin family, Roy grew up within the framework of orthodox
caste practices of his time: child-marriage, polygamy, sati and dowry were prevalent among the
higher castes and he had himself been married more than once in his childhood. The family’s affluence
had also made the best in education accessible to him.
● In 1814, he set up the Atmiya Sabha (or Society of Friends) in Calcutta to propagate the
monotheistic ideals of the Vedanta and to campaign against idolatry, caste rigidities, meaningless rituals
and other social ills. So, statement 1 is correct.
● Raja Rammohan Roy founded the Brahmo Sabha in August 1828; it was later renamed Brahmo
Samaj. Through the Sabha he wanted to institutionalize his ideas and mission.
● Rammohan Roy did not want to establish a new religion. He only wanted to purify Hinduism of the
evil practices which had crept into it. Roy’s progressive ideas met with strong opposition from orthodox
elements like Raja Radhakant Deb. So, statement 3 is not correct.

More details about Raja Ram Mohun Roy:


● Early Life and Education: Raja Ram Mohan Roy was born on May 22, 1772, in Bengal, India. He
received a traditional education in Sanskrit and Persian languages, and later, he learned English
and studied various religious texts and philosophies.
● Social Reforms: Raja Ram Mohan Roy was a strong advocate for social reforms in Indian society. He
actively worked against social evils like sati (the practice of widows self-immolating on their
husbands' funeral pyres) and fought for its abolition. He also campaigned for the rights of women,
education for all, and the eradication of caste discrimination.
● Founding of Brahmo Samaj: Raja Ram Mohan Roy founded the Brahmo Samaj in 1828, a
socio-religious reform movement that aimed to promote monotheism and reject idol worship and
other superstitious practices. Brahmo Samaj laid the foundation for modern Hindu reform movements
and contributed to the spread of rational thinking and progressive ideas.
● Tuhfat-ul-Muwahhidin. Among the works of Raja Rammohun Roy Tuhfatul Muwahhidin (Gift to the
Monotheists) occupies; a very important place. It is written in Persian with a preface in Arabic.
The rationalism of the Raja, without which Rammohun would not have been the cosmopolite and
Universalist that he was, free from ethnic, national and historic embodiments and limitation,
stands out prominently in the TuhfaL the Raja is un-understandable without his rationalism. So,
statement 2 is correct.
● Promoting Education: Raja Ram Mohan Roy emphasized the importance of education and the need
for modern scientific education in India. He established educational institutions like the Hindu
College in Calcutta (now Kolkata) and promoted the study of English, science, and other modern
subjects.
18

● Political Reforms: Raja Ram Mohan Roy advocated for political reforms and voiced his support for
representative government and the rule of law. He was critical of British colonial policies that he
perceived as exploitative and worked towards establishing a more just and equitable governance
system.
● Legacy: Raja Ram Mohan Roy is considered one of the key figures of the Bengal Renaissance and
a pioneer of Indian reform movements. His ideas and efforts laid the groundwork for social, religious,
and educational reforms that played a crucial role in shaping modern India.

Publications of Raja Ram Mohan Roy:

Work Year

Vedanta Gantha 1815

Kathopanishad 1817

A Conference between the Advocate for, and an Opponent of Practice of Burning Widows 1818
Alive (Bengali and English)

Mundaka Upanishad 1819

The Precepts of Jesus- The Guide to Peace and Happiness, A Defence of Hindu Theism 1820

Brahmanical Magazine 1821

Mirat Ul Akhbar 1822

Bengali Grammar 1826

History of Indian Philosophy, The Universal Religion 1829

Gaudiya Vyakaran 1833

Samvad Kaumudi 1836


Therefore, option (b) is the correct answer.

Q.16) With reference to Lord Metcalfe, consider the following statements:


1. The Press Act of 1835 signed by Metcalfe restored press freedom.
2. He was known as the "liberator of the Indian press"
Which of the statements given above is/are correct?

a) 1 only
b) 2 only
c) Both 1 and 2
d) Neither 1 nor 2

Answer: (c)
Explanation:
● Between 1822 and 1845, Charles Metcalfe was a British colonial administrator known as Sir Charles
Metcalfe.
● Sir Charles Metcalfe signed the Press Act, which restored press freedom, in 1835. So, statement 1
is correct.
● He was dubbed the "liberator of the Indian press" after overturning the heinous 1823 ordinance.
Under the new Press Act (1835), a printer/publisher was required to provide a detailed account of the
premises of a publication and to halt operations if a comparable declaration was submitted. So,
statement 2 is correct.
More about Lord Metcalfe:
19

● Governor-General of India: Lord Metcalfe was appointed as the Governor-General of India in 1835.
During his short tenure, he pursued policies aimed at liberalizing the British administration and
granting more autonomy to the Indian princely states. He advocated for a policy of non-intervention
and non-interference in the internal affairs of these states.
● Promotion of Education and Free Press: Lord Metcalfe was a strong supporter of education and
freedom of the press. He lifted restrictions on the Indian press and encouraged the growth of
vernacular newspapers, allowing for more open expression of ideas and opinions.
● Social Reforms: Lord Metcalfe showed a sympathetic approach towards social and religious customs in
India. He opposed the abolition of sati (widow burning), believing that social change should
come from within society rather than through external interference.
● Land Reforms: Lord Metcalfe initiated land reforms aimed at protecting the rights of landowners and
improving land revenue administration. He introduced measures to prevent the arbitrary seizure of
land by the East India Company.
● Return to Britain and Later Career: Lord Metcalfe's tenure as Governor-General was short-lived. He
returned to Britain in 1836 due to health reasons. However, he continued to be involved in public
service and held various high-ranking positions in the British government.

Therefore, option (c) is the correct answer.

Q.17) With reference to sangam period, consider the following pairs:


S. No. Terminology Meaning

1. Kurinchi The Hills

2. Palai The waste land

3. Mullai The forest

4. marutham The desert


How many pairs given above is/are not correctly matched?

a) One pair only


b) Two pairs only
c) Three pairs only
d) All the four pairs

Answer: (a)

Sangam period
● The period roughly between the 3rd century B.C. and 3rd century A.D. in South India (the area lying
to the south of river Krishna and Tungabhadra) is known as Sangam Period.
● It has been named after the Sangam academies held during that period that flourished under the royal
patronage of the Pandya kings of Madurai.
● At the sangams eminent scholars assembled and functioned as the board of censors and the choicest
literature was rendered in the nature of anthologies.
● These literary works were the earliest specimens of Dravidian literature.
● According to the Tamil legends, there were three Sangams (Academy of Tamil poets) held in the
ancient South India popularly called Muchchangam.
○ The First Sangam, is believed to be held at Madurai, attended by gods and legendary sages.
No literary work of this Sangam is available.
○ The Second Sangam was held at Kapadapuram, only Tolkappiyam survives from this.
○ The Third Sangam was also held at Madurai. A few of these Tamil literary works have
survived and are a useful source to reconstruct the history of the Sangam period.
Sangam literature:
● Sangam Literature is not religious in that sense.
● The poems belonging to what is called Sangam Literature are based on two main themes
20

○ love / akam,
○ heroism/ praise of Kings and their deeds/ puram.
● The literature of this period was comprised of the following types of works:
○ Ettuttokai ( Eight Anthologies)
○ Pattuppattu ( The Ten Long Poems)
○ Tolkappiyam ( A Grammar Treatise)
● The emotions portrayed in Sangam poetry are expressed through physical geography like the hills,
the seashore, the wasteland, the forest and the low land or the marshes.
● They are called Tenai and they form in the prosody analysis in the third section of the Tolkappiyam.
● The topographical related emotional Tenai’s in these poems include:
○ Kurinci (The Hills) → signifying lovers' meetings. So, pair 1 is correctly matched.
○ Neytal (The SeaShore) → signifying secret meetings at the sea shore and lovers' anxiety.
○ Palai (The Waste land) → signifying a lovers frustrating journey through the deserted
landscape. So, pair 2 is correctly matched.
○ Mullai (The Forest) → signifying happiness through physical and emotional union. So, pair 3
is correctly matched
○ Marutam (The Low land) → signifying a lover's unfaithful attitude. So, pair 4 is not correctly
matched.
Therefore, option (a) is the correct answer.

Q.18) With reference to different gharanas of Hindustani music, consider the following pairs:

Sl. No Gharanas Description

1. Dagari Gharana They incorporate a variety of layakari in their music.

2. Gwalior Gharana Known for its mastery of slow tempo ragas.

3. Agra Gharana Its compositions are a mix of Khayal and Dhrupad styles.
How many pairs given above is/are correct?

a) One pair only


b) Two pairs only
c) All three pairs
d) None of the above

Answer: (a)

Dhrupad
● It is one of the oldest and most grandiose forms of Hindustani classical music, and it is mentioned in
the Natyashastra (200 BC–200 AD).
● Dhrupad is said to have descended from earlier forms such as Prabhanda and Dhruvapada.
● The name is derived from the words 'dhruva' and 'pada,' and it refers to both the verse form of the
poetry and the style in which it is sung.
● Although Dhrupad became a classical form of music in the 13th century, it reached its pinnacle in the
court of Emperor Akbar.

Gharana Description

Dagari Gharana ● Dagar gharana sings in the dagar vani.


● This style places a strong emphasis on alap.
● Men in their family have trained and performed in pairs for several generations.
● Dagars are Muslims who sing Hindu texts about Gods and Goddesses.
21

● Example - Gundecha Brothers of Jaipur.

Darbhanga Gharana ● They perform the Khandar Vani and the Gauhar Vani.
● They place an emphasis on raga alap as well as composed songs over
improvised alap.
● They make it better by incorporating a variety of layakari. So, pair 1 is not
correctly matched.
● The Malliks are proponents of this school of thought.
● Ram Chatur Mallik, Prem Kumar Mallik, and Siyaram Tewari are currently
performing members.

Bettiah Gharana ● They perform the Nauhar and Khandar vani styles, each with their own set of
techniques that only those trained within the families are aware of.
● The Mishras are a well-known family who explain the system.
● Indra Kishore Mishra is the only living member who regularly performs.
● Furthermore, the Haveli style of Dhrupad is popular in the Bettiah and
Darbhanga schools.

Talwandi Gharana ● They sing the Khandar vani, but because the family is based in Pakistan, it is
difficult to keep it within the Indian music system.

Khayal
● 'Khyal' is a Persian word that means "idea or imagination."
● Amir Khusrau is credited with inventing this style. This form is popular among artists because it allows
for more improvisation.
● Khyal is built around a repertoire of short songs with two to eight lines. In general, a Khyal composition
is also known as a 'Bandish.'
● In the 15th century, Sultan Mohammad Sharqi was the most generous patron to Khyal.
● The use of taan in the composition is one of the most distinctive aspects of Khyal. As a result, Alap is
given much less space in Khyal music than Dhrupad.
● Two songs are typically used in a Khyal performance:
○ Bada Khyal - It is sung in slow-tempo
○ Chhota Khyal - It is sung in fast tempo
● Typically, the theme for these Khyal bandish is romantic. Even if they are related to divine creatures,
they sing about love.
● Outstanding Khyal compositions are written in praise of Lord Krishna.
● The major Gharanas of Khyal music are as follows:

Gharana Description

Gwalior Gharana ● It is one of the most ancient and elaborate Khyal Gharanas.
● It's very strict because there's equal emphasis on melody and rhythm.
● Despite the fact that the singing is very complex, they still prefer to perform
simple ragas.
● Nathu Khan and Vishnu Palushkar are two of the most well-known
expounders of this Gharana.

Kirana Gharana ● This gharana is named after the Uttar Pradesh town of Kirana.
● It was founded by Nayak Gopal, but Abdul Karim Khan and Abdul Wahid
Khan are credited with popularising it in the early twentieth century.
22

● The Kirana Gharana is well-known for its emphasis on precise tuning and note
expression.
● The Kirana gharana is known for its mastery of slow tempo ragas. So,
pair 2 is not correctly matched.
● They place a greater emphasis on the composition's melody and the clarity of
the text's pronunciation in the song.
● They also favour using traditional ragas or the Sargam.

Agra Gharana ● Haji Sujan Khan, according to musicologists, founded this Gharana in the
19th century, while historians believe Khuda Baksh did.
● Faiyaz Khan revitalised the Gharana by adding a fresh and lyrical touch to it.
● It has since been renamed as Rangeela Gharana.
● Agra Gharana compositions are a mix of Khyal and Dhrupad styles. So,
pair 3 is correctly matched.
● Bandish is given special attention by the artists in the composition.
● Mohsin Khan Niazi and Vijay Kichlu are two prominent exponents of this
school of thought at the moment.

Patiala Gharana ● The Gharana was founded in the nineteenth century by Bade Fateh Ali Khan
and Ali Baksh Khan.
● The Maharaja of Patiala in Punjab provided the initial funding. They quickly
established a reputation for ghazal, thumri, and khayal.
● They emphasise the importance of using more rhythm.
● Because their compositions emphasise emotions, they frequently employ
ornamentation or alankaras in their music.
● Bade Ghulam Ali Khan Sahab, one of India's greatest Hindustani classical
vocalists who bridged the gap between vocals being limited to an elite
audience, is the most well-known composer from this Gharana.
● He was famous for his rendition of Raga Darbari.
● The Gharana is distinct because it employs distinctive taans, gamak, and
gayaki.

Bhendibazaar ● It was founded in the nineteenth century by Chhajju Khan, Nazir Khan, and
Gharana Khadim Hussain Khan.
● They grew in popularity and fame as the singers were trained to hold their
breath for extended periods of time.
● These musicians were able to sing long passages in a single breath by
employing this technique.
● Furthermore, they are distinct in that they include some Carnatic ragas in their
enviable repertoire.

Therefore, option (a) is the correct answer.


23

Q.19) The Ashokan edicts are located in which of the following locations?
1. Sanchi
2. Champaran
3. Amritsar
4. Khyber Pakhtunkhwa
5. Udupi
Select the correct answer using the codes given below:

a) 1, 2 and 4 only
b) 2, 4 and 5 only
c) 1, 2 and 5 only
d) 2, 3 and 4 only

Answer: (a)

Pillar Edicts of Ashoka


● There are many pillar edicts.
● All of them are stone monoliths.
● Two types of stones were used to carve the pillars – white and buff coloured sandstone and
quartzite.
● These pillars were found in Sanchi (Madhya Pradesh), Delhi, Sarnath and Allahabad (Uttar Pradesh),
Kandahar (Afghanistan), Champaran (Bihar), Mansehra - Khyber Pakhtunkhwa (Pakistan), and
Amaravati (Andhra Pradesh). So, statements 1, 2 and 4 are correct.
● On top of the pillars, symbols of bulls, horses, or lion (facing all four directions) were crafted.
● Pillar edicts are 40-50 feet high and weigh approximately 50 tons.
Description of Some of the edicts
● Edict I: Ashoka’s postulates about guarding his people.
● Edict II: Dhamma
● Edict III: Circumvent exercises of brutality, pride, sin, harshness and rage among his followers.
● Edict IV: Duties of the Rajukas.
● Edict V: Record of animals and birds that should not be sacrificed on particular days.
○ A separate list mentioning animals that should never be killed.
● Edict VI: Dhamma policy of the kingdom.
● Edict VII: Ashoka’s work for attaining Dhamma. Developing a tolerance for all religions.
Therefore, option (a) is the correct answer.

Q.20) With reference to various folk dances of India, consider the following pairs:
S. No. Folk dance Region

1. Tarangamel Goa

2. Matki Madhya Pradesh

3. Danda-Jatra Odisha

4. Biraha Uttar Pradesh


How many pairs given above is/are correctly paired?

a) Only one pair


b) Only two pairs
c) Only three pairs
d) All the four above

Answer: (c)
24

Biraha
● Region: Bihar
● It depicts the anguish of women whose partners are gone from home.
● This dance form, on the other hand, is only performed by guys who also play feminine characters. So,
pair 4 is not correctly matched.
Danda-Jatra
● Region: Odisha
● It's a one-of-a-kind combination of dance, drama, and music.
● While it mostly tells Shiva myths and legends, the overall message is social harmony and brotherhood.
So, pair 3 is correctly matched.
Matki
● Region: Madhya Pradesh
● Matki is a traditional dance performed by women in the Malwa region on special occasions such as
weddings and other celebrations.
● It's mostly done solo, with a number of earthen pots balanced on the head.
● Aada and Khada Nach are two prominent Matki dance variations. So, pair 2 is correctly matched.
Tarangamel
● Region: Goa
● It is a Goan folk dance that celebrates the region's youthfulness.
● It is practiced during the Hindu festivals of Dussehra and Holi.
● It's a visual show, with rainbow-like outfits, multi-colored banners, and streamers. So, pair 1 is
correctly matched.
Therefore, option (c) is the correct answer.

Q.21) With reference to the Folk Theatre of India, consider the following statements:
1. Ankia Naat is an Assamese drama founded by the Vaishnava Saint Sankaradeva.
2. Ramlila is a performance of the Mahabharata that takes place in the days leading up to Dussehra.
Which of the statements given above is/are correct?

a) 1 only
b) 2 only
c) Both 1 and 2
d) Neither 1 nor 2

Answer: (a)

Ankia Naat
● It is an Assamese traditional one-act drama, founded in the 15th-16th century AD by the legendary
Vaishnava Saint Sankaradeva and his disciple Madhavdev. So, statement 1 is correct.
● It is staged in the manner of an opera and covers events from Lord Krishna's life.
● The Sutradhar, or narrator, is accompanied by Gayan-Bayan Mandali, a group of musicians who play
the 'khol' and cymbals.
● The usage of masks to represent particular expressions is one of the distinguishing traits of this kind of
theatre.
● The plays are written in Brajavali, an Assamese-Maithili hybrid language.

Ramlila
● Ramlila is a well-known folk theatre in the Uttar Pradesh area.
● It is a performance of the Ramayana that includes music, dances, and conversations that take place in
the days leading up to Dussehra. So, statement 2 is not correct.
● It is usually performed by male performers who also play the part of Sita.
● The Ramlila festivities were declared by UNESCO as one of the "Intangible Cultural Heritage of
Humanity" in 2008.

Therefore, option (a) is the correct answer.


25

Q.22) Consider the following statements regarding the Maratha Confederacy:


1. It was started by Peshwa Madhavrao I.
2. The regions under confederacy were to be ruled in the name of Peshwa.
3. The death of Madhavrao I in 1772 weakened the control of the Peshwas over the confederacy.
Which of the statements given above are correct?

a) 2 only
b) 1 and 3 only
c) 3 only
d) 1, 2, and 3

Answer: (c)

● Bajirao I (1720-40) had started a confederacy of prominent Maratha chiefs to manage the rapidly
expanding Maratha power, and to some extent appease the kshatriya section of the Marathas as
Peshwas were brahmins. So, statement 1 is not correct.
● Under the arrangement of the Maratha confederacy, each prominent family under the chief was
assigned a sphere of influence which he was supposed to conquer and rule. It was to be ruled in
the name of the Maratha king and not in the name of Peshwa.
○ The Maratha families which emerged prominent were
■ the Gaekwad of Baroda,
■ the Bhonsle of Nagpur,
■ the Holkars of Indore,
■ the Sindhias of Gwalior, and
■ the Peshwa of Poona. So, statement 2 is not correct.
● The confederacy, under Bajirao I to Madhavrao I worked cordially but the Third Battle of Panipat
(1761) changed everything.
○ The defeat at Panipat and later the death of the young Peshwa, Madhavrao I, in 1772,
weakened the control of the Peshwas over the confederacy. Thus, in the second half of the
18th century C.E, the control of Peshwa was weakened over the confederacy. So, statement 3
is correct.
Therefore, option (c) is the correct answer.

Q.23) With reference to Pallava ruler NarasimhaVarman-I, consider the following statements:
1. During his reign the Pancha Ratha Temples were constructed.
2. Chinese traveler Hiuen Tsang visited Kanchipuram during the rule of Narasimhan I.
3. Nayanar saints like Appar, Siruthondar and Tirugnanasambandar were contemporaries of Narasimhan I
Which of the statements given above is/are correct?

a) 1 and 3 only
b) 2 and 3 only
c) 1 and 2 only
d) 1, 2, and 3

Answer: (d)
Explanation:

● Narasimhan I / NarasimhaVarman-I was an


emperor of the Pallava dynasty who ruled South
India from 630 CE – 668 CE.
● He shared his father Mahendravarman's love of
art and completed the work started by
Mahendravarman in Mamallapuram.
● Considered the greatest of the Pallavas. Also
called Narasimhavarman Mahamalla/Mamalla.
● He founded the city of Mamallapuram or
Mahabalipuram which is named after him.
● Defeated and killed Pulakesin II in 642 AD.
26

● He took control of Vatapi, the Chalukya capital and assumed the title ‘Vatapikonda’.
● He sent a naval expedition to Sri Lanka and reinstated the Sinhalese Prince Manivarma.
● During his reign, the famous Pancha Rathas Temple was constructed which is Rock Cut Temple, a
UNESCO World Heritage Site. So, statement 1 is correct.
● It was during his reign, in 640 CE, that the Chinese traveler Hiuen Tsang visited Kanchipuram. So,
statement 2 is correct.
● Narasimhan I was a devotee of Shiva. The great Nayanar saints like Appar, Siruthondar and
Tirugnanasambandar lived during his reign. So, statement 3 is correct.
● He was succeeded by his son Mahendravarman II who ruled from 668 to 670 AD.
Therefore, option (d) is the correct answer.

Q.24) In the context of the history of India, which of the following best describes the term ‘Kuta’,’Ayatasra’,
‘Gaja-Prishta’,’vrittayata’?

a) Irrigation system during Chola period


b) Name of coin during the Vijayanagara period
c) Subdivisions of Dravida temples
d) Ancient books on Astronomy

Answer: (c)

Dravida style of temple architecture

● The Dravidian style of temple architecture of


South India was pioneered by the Pallavas who
reigned in parts of Karnataka, Andhra Pradesh,
and northern Tamil Nadu until the ninth century.
● The early buildings are generally attributed to
the reign of Mahendravarman I, a
contemporary of the Chalukyan king, Pulakesin II
of Karnataka. Narasimhavarman I, also known as
Mamalla, who acceded the Pallava throne around
640 CE, is celebrated for his architectural works.
Main features of this style of temple architecture:
● The Dravida temple is enclosed within a
compound wall.
● The front wall has an entrance gateway in its centre, which is known as a Gopuram.
● The shape of the main temple tower known as vimana in Tamil Nadu is like a stepped pyramid that
rises up geometrically rather than the curving shikhara of North India.
● In the South Indian temple, the word ‘shikhara’ is used only for the crowning element at the top of
the temple which is usually shaped like a small stupika or an octagonal cupola— this is equivalent to
the amalak and kalasha of North Indian temples.
● Fierce Dvarapalas or the door-keepers guarding the temple adorn the entrance to garbhagriha
● It is common to find a large water reservoir, or a temple tank, enclosed within the complex.
● At some of the most sacred temples in South India, the main temple in which the garbhagriha is
situated has, in fact, one of the smallest towers. This is because it is usually the oldest part of the
temple.
Classification of Dravidian temples:
● Just as there are many subdivisions of the main types of Nagara temples, there are subdivisions also
of Dravida temples.
● These are basically of five different shapes: square, usually called kuta, and also caturasra;
rectangular or shala or ayatasra; elliptical, called Gaja-Prishta or elephant-backed, or also called
vrittayata, deriving from wagon-vaulted shapes of apsidal chaityas with a horse-shoe shaped entrance
facade usually called a nasi; circular or vritta; and octagonal or ashtasra
● The above classification is a simplistic one since several different shapes may be combined in specific
periods and places to create their own unique style.
Therefore, option (c) is the correct answer.
27

Q.25) With reference to Buddhism Mudras, consider the following statements:


1. Bhumisparsa Buddha symbolizes the moment when Buddha attained enlightenment under the Bodhi
tree.
2. Dharmachakra Mudra symbolizes protection, peace, and the dispelling of fear.
3. Vitarka Mudra symbolizes the transmission of the dharma or the truth teachings of the Buddha.
Which of the statements given above is/are correct?

a) 1 and 3 only
b) 2 and 3 only
c) 1 and 2 only
d) 1, 2 and 3

Answer: (a)

● Mudras are a non-verbal mode of communication and self-expression, consisting of hand gestures
and finger postures.
● Many such hand positions were used in the Buddhist sculpture and painting of India, Tibet, China,
Korea and Japan.
● Ranging from the poetic and complicated Dharmachakra mudra to the prosaic, universal prayer
symbol, mudras reference anecdotes from the historical Buddha’s life and convey elements of
Buddhist practice like meditation and teaching.
Different Mudras of Buddha
Mudra Image

Dharmachakra Mudra
● Dharmachakra in Sanskrit means the 'Wheel of Dharma'.
● It symbolizes the first sermon of Buddha. So, statement 2
is not correct.

Bhumisparsha Mudra
● It means 'touching the earth'.
● Symbolizes the moment when Buddha attained
enlightenment under the Bodhi tree. So, statement 1 is
correct.
28

Varada mudra
● Also known as favorable mudra, it is a gesture of granting
wishes or mercy

Dhyana Mudra
● It is the mudra of meditation, of concentration on the Good
law, and of the attainment of spiritual perfection.

Abhaya Mudra
● It symbolizes protection, peace, and the dispelling of fear.
So, statement 2 is not correct

Vajra Mudra
● This gesture represents the five elements of air, water, fire,
earth, and metal, which are represented by the blazing
thunderbolt.

Vitarka Mudra
It is used to symbolize the transmission of the dharma or the truth
teachings of the Buddha. So, statement 3 is correct.
29

Jnana Mudra
Jnana mudra is a psychic gesture of knowledge.

Karana Mudra
In Buddhism, karana mudra is believed to remove obstacles, cultivate
inner peace and guide the practitioner towards enlightenment.

Uttarabodhi Mudra
It is mostly known as the mudra for enlightenment

Anjali Mudra
Anjali mudra is used as a posture of composure, of returning to one's
heart, whether you are greeting someone or saying goodbye,
initiating or completing an action.

Therefore, option (a) is the correct answer.


30

Q.26) With reference to architecture of jagannath Temple, consider the following statements:
1. The temple has four distinct components namely Deula, Mukhmandir, NataMandapa and Bhoga
Mandapa.
2. The temple is built in the Vesara style of temple architecture.
3. The crowning element of the temple is an eight wheel Chakra made of ashtadhatu.
4. There are four entry gates to the temple.
Which of the statements given above is/are correct?

a) 1 and 3 only
b) 1, 3 and 4 only
c) 1 and 4 only
d) 1, 2, 3 and 4

Answer: (b)

Jagannath temple:
● The temple was rebuilt by the Ganga dynasty king Anantavarman Chodaganga in the 12th
century CE, as suggested by the Kendupatna copper-plate inscription of his descendant
Narasimhadeva II.
● The temple complex was further developed during the reigns of the subsequent kings, including those of
the Ganga dynasty and the Gajapati dynasty.
● The deities within the shrine are believed to be much older and are associated with the great
mythical ruler of the Satya-yuga, King Indrayumna, nephew of Lord Ram.
The architecture of the temple
● The Jagannatha temple is one the most refined pieces of Kalingan Architecture.
● The temple stands in the center of an inner courtyard on a high-raised platform.
● It has four components:
○ Vimana or Deula (Garbhagriha)
○ Mukh Mandir / Jagamohana
○ Nata Mandapa
○ Bhoga Mandapa. So, statement 1 is correct.
● The architectural style of the Jagannath temple is a combination of two types-Rekha and Pidha. Both
are regional variations of Nagara style of architecture. So, statement 2 is not correct.
○ The vimana is built in the nagara style Rekha deula.
○ The jagamohana is in the Pidha deula style.
● The main temple, the sanctuary (Garbhagruha) is commonly known as Vimana or deula and is built in
the framework of Nagara Sthaptya (architecture) as a Rekha deula characterized by a curvilinear
tower known as sikhar.
● The crowning at the top is the ‘Neelachakra’ (an eight-spoked wheel) of Lord Vishnu. It is made
out of Ashtadhatu and is considered sacrosanct. So, statement 3 is correct.
● The viman or deula is built on the Pancha Ratha ground plan in such a manner that its vertical
shape starting from the bottom up to the highest level of the curvilinear shikhara is divided into five
projected column or pillar-like structure i.e rath in each of the four sides.
● Here lies the unique architectural quality of the Pancha Ratha Rekha deula of Shree Jagannatha.
● Both the Vimana (Sanctuary) and the porch (Jagamohana) are divided into five principal parts
along the vertical plane through the foundation:
○ pishta or pitha
○ the bada (vertical or perpendicular wall)
○ gandi (trunk of a body) i.e, the curvilinear tower in the case of a Rekha deula.
○ pyramidal roof in the case of pidha deula
○ mistake (head of the crowning elements)
31

● The Natamandap (the audience hall) and the Bhoga Mandap (the hall for residuary offerings) are built
in a row in an axial alignment in an east-west direction.
● There are four gates to the temple- Eastern ‘Singhdwara’ which is the main gate with two crouching
lions, Southern ‘Ashwadwara’, Western ‘Vyaghra Dwara, and Northern ‘Hastidwara’. There is a
carving of each form at each gate. So, statement 4 is correct.
Therefore, option (b) is the correct answer.

Q.27) Which of the following statements is/are correct with respect to the city of Fatehpur Sikri?
1. It was founded by Jalaluddin Mohammad Akbar in honour of Sheikh Salim Chisti.
2. The Diwan-i-Am was used for celebrations and public prayers.
3. The Daulat khana-i-khas was the emperor's private chamber connected to the palaces of the queens.
Which of the statements given above is/are correct?

a) 1 and 2 only
b) 1 and 3 only
c) 2 and 3 only
d) 1, 2 and 3

Answer: (d)

Fatehpur Sikri:
● Fatehpur Sikri, a city predominantly in red sand-stone,
situated at a distance of 37 kms from Agra, was built by
the Mughal Emperor Jalal-ud-din Mohammad Akbar, in
honour of the great Sufi saint Sheikh Salim Chisti. So,
statement 1 is correct.
● Its magnificence and uniqueness offers a fine example of
the emperor's architectural finesse.
● Akbar's tolerant religious views and interest in literature,
architecture and fine arts gave the buildings at Fatehpur
Sikri a charismatic blend of Islamic and Hindu
elements in their style and design.
● Fatehpur Sikri is enclosed by a 11 kms long
fortification wall interspersed with numerous gateways.
32

● He had planned this city as his capital but shortage of water compelled him to abandon the city.
○ After this, within 20 years, the capital of Mughals was shifted to Lahore.
● Fatehpur Sikri is the best example of the culmination of Hindu and Muslim architecture.
● Fatehpur Sikri Mosque is said to be a copy of the mosque in Mecca and has designs, derived from the
Persian & Hindu architecture.
Important buildings in the city:
Diwan-I-Am
● The journey to the royal palace begins with Diwan-I-Am or the Hall Of Public Audience.
● This hall was also used for celebrations and public prayers. So, statement 2 is correct.
● It has cloisters on three sides of a rectangular courtyard.
Diwan-khana-I-khaas
● To the right is an apparently looking two
storeyed building, with corner kiosks, known
as diwan-khana-I-khaas or Hall Of Private
Audience.
● On entering it, one finds only a single vaulted
chamber. In the centre stands a profusely carved
column supporting a collosal-bracketed capital.
● Four narrow causeways project from the centre
and run to each corner of the chamber.
● It is believed that Akbar’s throne occupied the
circular space over the capital and the corners
were assigned to the four ministers.
Daulat khana-I-khas:
● Located in the corner to the left is the emperor’s
private chamber. It has two main rooms on
the ground floor.
● One housed Akbar’s library while the larger room was his resting area.
● On the first floor is the Khwabgah or the bed-chamber.
● It was connected with the Turkish Sultana’s house, the Panch Mahal, Mariam’s House and the
Jodha Bai’s palace by corridors. So, statement 3 is correct.
Palace of Jodha Bai:
● To the left of the Sunehra Makan is the largest and the most important building in the royal palace,
named after Akbar’s Rajput wife, Jodha Bai.
● This spacious palace was assured of privacy and security by high walls and a 9 metre guarded gate to
the east.
● The architecture is a blend of styles with Hindu columns and Muslim cupolas.
Hawa Mahal And Nagina Masjid
● This small-screened wind tower faces the garden and is attached to the palace.
● The garden is laid out in the Char Bagh style with straight walls intersecting at right angles and
divided by shallow channels.
Buland Darwaza
● This gate can be approached from the outside by a 13-metre flight of steps which adds to its
grandeur.
● The gate erected in 1602 AD to commemorate Akbar’s victory over Deccan is the highest and
grandest gateway in India and ranks among the biggest in the world.
Therefore, option (d) is the correct answer.

Q.28) National Monuments Authority has recommended Sankalp Bhumi Banyan tree campus in Vadodara to be
declared as Monuments of National Importance. It is associated with which of the following personalities?

a) V. D. Savarkar
b) Lala Lajpat rai
c) Sardar Vallabh Bhai Patel
d) B. R. Ambedkar

Answer: (d)
33

● National Monuments Authority has recommended following two sites to be declared as Monuments
of National Importance:
○ Sankalp Bhumi Banyan tree campus in Vadodara, where Dr. Ambedkar had taken a
resolve to eradicate untouchability on 23rd September 1917.
○ PratapRao Bhosale High School, place in Satara in Maharashtra where Dr Ambedkar
received his primary education. So, option (d) is correct.
Dr. B. R. Ambedkar
● Dr. Bhimrao Ramji Ambedkar, popularly known as Babasaheb Ambedkar, was one of the main
architects of the Indian Constitution.
● He was a very well known political leader, philosopher, writer, economist, scholar and a social
reformer who dedicated his life to eradicating untouchability and other social inequality in India.
● He was born on 14 April 1891 in Madhya Pradesh in Hindu Mahar Caste. He had to face severe
discrimination from every corner of the society as the Mahar caste was viewed as "untouchable" by
the upper class.
● He was popularly known as Babasaheb. He was an Indian jurist, economist, politician, and social
reformer who Chaired the Drafting Committee of the Constituent Assembly and was India’s First
Minister for Law and Justice.
● He is known as the maker and conscience keeper of modern India.
Therefore, option (d) is the correct answer.

Q.29) With reference to the Battle of Saraighat, consider the following statements:
1. It was fought between the British Empire and the Ahom Kingdom.
2. It was fought on the Brahmaputra River.
3. Lachit Borphukan is known for his leadership in Battle of Saraighat
Which of the statements given above is/are correct?

a) 1 and 2 only
b) 2 and 3 only
c) 1 and 3 only
d) 1, 2 and 3

Answer: (b)

Battle of Saraighat:
● The Battle of Saraighat was a naval battle fought in 1671 between the Mughal Empire (led by the
Kachwaha raja, Ram Singh I), and the Ahom Kingdom (led by Lachit Borphukan) on the
Brahmaputra river at Saraighat, now in Guwahati, Assam, India. So, statement 1 is not correct and
2 is correct.
● Lachit Borphukan is known for his leadership in the Battle of Saraighat. He was the Commander in
Chief of the Ahom Kingdom Army. So, statement 3 is correct.
● Although weaker, the Ahom Army defeated the Mughal Army by clever diplomatic negotiations to
buy time, guerrilla tactics, psychological warfare, military intelligence and by exploiting the sole
weakness of the Mughal forces—its navy.
● The Battle of Saraighat was the last battle in the last major attempt by the Mughals to extend their
empire into Assam.
The aftermath of the Battle of Saraighat
● Lachit Borphukan passed away from natural causes in 1672, a year after the battle of Saraighat.
● Although the engagement was a decisive one, it did not end the Ahom-Mughal conflicts.
● Guwahati would be taken by the Mughals when it was abandoned in 1679 by Lachit’s successor Laluk
Sola.
● But it would fall back into the hands of the Ahom Kingdom once again when the Ahom Army under
Dihingia Alun Borbarua would defeat the Mughals at the battle of Itakhuli in 1682. This battle would
permanently end Mughal presence in Assam and no further campaigns would be undertaken by
them against the Ahom Kingdom
Therefore, option (b) is the correct answer.
34

Q.30) Consider the following pairs:


S. No. Commission Major recommendation

1. Sadler Commission All universities to be freed from excessive official control.

2. Raleigh Commission Technical and commercial education by universities.

3. Hunter Commission Instruction at primary level to be given in mother tongue.

4. Hartog Commission Universities have the power to appoint their own staff.
How many pairs given above is/are correctly matched?

a) One pair only


b) Two pairs only
c) Three pairs only
d) All the four pairs

Answer: (b)

Sadler Commission(1917)
● The Sadler Commission was a commission set up to inquire into the conditions and prospects of
the University of Calcutta.
● Some of the main recommendations of the Sadler Commission were as follows:
○ All the teaching staff in Calcutta were to be organized so that Calcutta University would be
a full-fledged university
○ Separate residential and teaching university was to be set up at Dacca
○ All universities were to be freed from all excessive official controls and government
interference in academic matters must be stopped. So, pair 1 is correctly matched.
Raleigh Commission(1902)
● Sir Thomas Raleigh established the Raleigh Commission on January 27, 1902, with the goal of
examining the present and future of Indian institutions and making suggestions for enhancing
their organization and performance.
● A report on either primary or secondary education was not possible for the Commission.
● The main objective of the Act was to raise the bar for the system and improve the standard of
education in India.
● The Indian Universities Act was enacted in 1904 as a result of the Commission’s report and
recommendations.
● Major recommendations:
○ Improvement in the overall condition of education and upgrade the system to a better
level.
○ The universities will have powers to appoint their own staff including the academic staff.
So, pair 2 is not correctly matched.
○ The number of fellows was limited to 50-100.
○ However, for Bombay, Madras, and Calcutta Universities, the number of elected Fellows was
fixed at 20.
Hunter Commission:
● Recommendations:
○ At the high school level, there should be two types of education arrangements- vocational and
business education.
○ Private stakeholders are welcomed especially in primary education.
○ The medium of instruction in primary education should be the mother tongue. So, pair 3
is correctly matched.
○ It is the sole responsibility of the government to spread primary education among the
tribal and backward people.
Hartog Commission:
● Recommendations:
○ The duration of the primary course was fixed to 4 years.
35

○ The government will promote technical and commercial education by universities to


control the problem of unemployment in India. So, pair 4 is not correctly matched
Therefore, option (b) is the correct answer.

Q.31) With reference to Quit India Movement, consider the following pairs:
Sl. No Parallel Governments Place/Region

1. Prati Sarkar Tamluk

2. Jatiya Sarkar Satara

3. Nyayadan Mandals Bombay

4. Chasi Maulia Talcher


How many pairs given above is/are correct?

a) One pair only


b) Two pairs only
c) Three pairs only
d) All the four pairs

Answer: (a)

● The Quit India Movement, also known as the August Movement or August Kranti, was a rallying call
issued by Mahatma Gandhi from the Bombay session of the All-India Congress Committee in
Mumbai on August 8, 1942.
● Many nationalists fled to the underground and engaged in subversive activities.
● Socialists, Forward Bloc members, Gandhi ashramites, revolutionary nationalists, and local
organizations from Bombay, Poona, Satara, Baroda, and other parts of Gujarat, Karnataka, Kerala,
Andhra Pradesh, United Provinces, Bihar, and Delhi took part in these activities.
● Rammanohar Lohia, Jayaprakash Narayan, Aruna Asaf Ali, Usha Mehta, Biju Patnaik,
Chhotubhai Puranik, Achyut Patwardhan, Sucheta Kripalani, and R.P. Goenka were among the key
figures involved in underground activity.
● Usha Mehta founded an underground radio station in Bombay.
● This phase of underground activity was intended to maintain popular morale by maintaining a line of
command and guidance for the distribution of arms and ammunition.

Parallel governments were established in many places, including

● Ballia (for a week in August 1942) under Chittu Pandey. Many members of Congress were released as
a result of his efforts.
● Tamluk (Midnapore, from December 1942 to September 1944) - Jatiya Sarkar worked on cyclone
relief, sanctioned school grants, distributed paddy from the rich to the poor, organized Vidyut Vahinis,
and so on. So, pair 1 is not correctly matched.
● Satara (mid-1943 to 1945) - dubbed "Prati Sarkar," it was organized by leaders such as Y.B. Chavan,
Nana Patil, and others. Village libraries and Nyayadan Mandals were established, as were prohibition
campaigns and 'Gandhi marriages.' So, pairs 2 and 3 are not correctly matched.
● Businessmen (through donations, shelter, and material assistance), students (as couriers), simple
villagers (by refusing to provide information to authority), pilots and train drivers (by delivering bombs
and other material), and government officials, including police, all provided active assistance (who
passed on secret information to the activists).
● Talcher Parallel government:
○ The Quit India movement had its flame in the princely state of Talcher.
○ It had already witnessed the struggle against forced labour (Bethi) forest laws and autocratic
rule in September 1938.
36

○ The immediate cause of the popular upsurge was a rumour that Pabitra Mohan Pradhan,
President of Talcher State Prajamandal had been murdered.
○ For all practical purposes, the State's administration collapsed on 31st August 1942.
○ The Jatiya Sarkar was called 'Chasi Maulia' or 'Mazdoor Raj'. It was to be set up on the basis
of an adult franchise in each village, block, circle, pargana, and sub-division. So, pair 4 is
correctly matched.
○ The Central Government was accordingly constituted on the same line. Some government
servants voluntarily resigned, burnt their European dress and uniforms, set fire to the official
records, and swore allegiance to the New Raj.

Therefore, option (a) is the correct answer.

Q.32) With reference to Provisional government of Free India, consider the following statements:
1. It was established in Singapore by Subhash Chandra Bose.
2. It meant to support the axis powers in the second world war against Britain.
3. During the war, it nominally controlled Andaman and Nicobar and the entire north-east region of India.
Which of the statements given above is/are correct?

a) 1 and 3 only
b) 2 and 3 only
c) 1 and 2 only
d) 1, 2 and 3

Answer: (c)

Provisional Government of Free India:


● On 21 October 1943, the Provisional Government of Free India was established as a
government-in-exile in Singapore by Indians in exile headed by Subhash Chandra Bose. So,
statement 1 is correct.
○ It was also called Azad Hind or Free India.
● Azad Hind was established as part of a political movement originating outside India by Indians
against British imperialism.
● Subhash Chandra Bose was the most prominent leader of this movement and he was also the Head of
State of the provisional government.
● It was established with financial, political and military support from Imperial Japan.
○ This political movement was spearheaded by Bose, who was called Netaji by the Azad Hind
members, and it meant to take sides with the Axis powers (including Nazi Germany, Italy
and Japan) in order to wrest India of British rule (who were part of the Allied Forces during
World War II). So, statement 2 is correct.
● Azad Hind staked claim to the entire subcontinent as its own, but in actuality, it governed Indian
military and civilian personnel in the Southeast Asian British colonial territory.
○ After the Andaman and Nicobar Islands, and parts of Nagaland and Manipur (not the
entire North-east) were captured by Japan in 1943 as part of the ongoing war, the nominal
authority of these regions was given to Azad Hind. so, statement 3 is not correct.
○ These islands were renamed ‘Shaheed’ and ‘Swaraj’.
● While some historians say that the Azad Hind was an independent government, most say that it
was dependent on the Japanese.
○ The provisional Indian government was recognised only by the Axis powers and their allies.
● The Azad Hind government had its own court, civil code and currency.
● Its provisional capital was Port Blair, while its capital-in-exile was Rangoon and Singapore.
○ Subhash Chandra Bose was the Prime Minister.
○ S. A. Ayer was the Minister of Broadcasting and Publicity
○ Lt. Col. A. C. Chatterji was the Minister of Finance.
● There were a number of secretaries and advisors also.
● The British were successful in defeating the INA and ousting the Japanese from the Andaman and
Nicobar Islands and the northeast.
● Azad Hind was disestablished on 18 August 1945.
Therefore, option (c) is the correct answer.
37

Q.33) Consider the following statements with reference to the events related to Gandhiji in South Africa:
1. Gandhi organised the first Satyagraha campaign to protest against the Transvaal Asiatic ordinance.
2. He fought against the nullification of non christian marriages.
3. He established Phoenix farm and Tolstoy farm in South Africa.
Which of the statements given above is/are correct?

a) 1 and 3 only
b) 2 and 3 only
c) 1 and 2 only
d) 1, 2 and 3

Answer: (d)

● Gandhi raised political awareness through regular comments in Indian Opinion (his newspaper) and
petitions to the governments of Natal, India, and Britain.
● Gandhi formed the Natal Indian Congress in 1894. This organisation led nonviolent protests against
white people's oppressive treatment of native Africans and Indians.
● In 1896, he visited India briefly and gathered 800 Indians to serve alongside him in South Africa. An
enraged mob greeted them, and Gandhi was injured in the attack.
● During the outbreak of the Boer War in 1899, Gandhi gathered approximately 1,100 Indians and
organised the Indian Ambulance Corps for the British, but ethnic discrimination and torture against
Indians persisted.
● Gandhi was inspired by English artist John Ruskin's book Unto This Last, and he established
Phoenix Farm near Durban.
○ Gandhi would come here to train his cadres in nonviolent Satyagraha, or peaceful restraint.
Satyagraha is said to have begun at Phoenix Farm.
● Satyagraha, on the other hand, was shaped into a weapon of protest at the Tolstoy Farm, Gandhi's
second camp in South Africa. So, statement 3 is correct.
● Gandhi organised the first Satyagraha campaign in September 1906 to protest the Transvaal
Asiatic ordinance, which was enacted against the local Indians. In June 1907, he held another
Satyagraha against the British. So, statement 1 is correct.
● He was imprisoned in 1908 for organising nonviolent movements. He was released, however, after
meeting with General Smuts, a British Commonwealth statesman.
● However, he was later attacked for this and sentenced to prison again, prompting him to organise
Satyagraha once more.
● He was also in long-term negotiations with the Attorney-General of Transvaal, Jan Smuts, first on
behalf of Indians in that Province, and later, after the Union was established in 1910, on behalf of all
South African Indians.
● He was sentenced to three months in prison in Volkshurst and Pretoria in 1909. Following his release,
Gandhi travelled to England to seek the help of the Indian community there.
● In 1913, he also fought against the nullification of non-Christian marriages. So, statement 2 is
correct.
● Gandhi organised yet another peaceful resistance campaign in Transvaal against the oppression of
Indian minorities. He led a group of approximately 2,000 Indians across the Transvaal border.
● Gandhi spent a total of 21 years in South Africa. By the end of his stay, the government had passed the
Indian Relief Act, which granted many of Gandhi's and his colleagues' demands.

Therefore, option (d) is the correct answer.


38

Q.34) Which of the following was/were the works of Dhondo Keshav Karve?
1. Establishment of widow marriage organization in 1893.
2. Established Hindu Widow’s Home in 1896.
3. Established the first women university in India in 1916.
4. Wrote two autobiographies namely ‘Atmawrutta’ and ‘Looking Back’.
Select the correct answer using the codes given below:

a) 1 and 3 only
b) 1 and 4 only
c) 2 and 4 only
d) 1, 2, 3 and 4

Answer: (d)

Dhondo Keshav Karve:


● On April 18, 1858, Dr. Dhondo Keshav Karve was born
in Ratnagiri, Maharashtra.
● Dhondo Keshav Karve, better known as Anna Karve,
was a pioneer in empowering women and promoting
widows' education in pre-independence India.
Works of Karve:
● The social reformer and educator established the
Widow Marriage Association in 1893. So, statement
1 is correct.
● Maharishi Karve founded an educational institution,
Hindu Widows Home in 1896, in Poona to help widows
support themselves, in case they were unable to
remarry. So, statement 2 is correct.
● After reading a book on the Japanese Women's
University in Tokyo, sent to him by his friends,
Maharshi Karve established India's first university for women in 1916. So, statement 3 is correct.
○ Initially, only five students got themselves enrolled.
○ He started the Shreemati Nathibai Damodar Thackersey Women's University in 1916.
● He wrote two autobiographies, ‘Atmawrutta' in Marathi (1928) and ‘Looking Back' in English (1936).
So, statement 4 is correct.
● Dhondo Keshav Karve died in Pune, India, on November 9, 1962, at the age of 104.
Therefore, option (d) is the correct answer.

Q.35) With reference to sessions of Indian National Congress (INC) and its Presidents, consider the following
pairs:
S. No. Sessions of INC President

1. Gaya session (1922) C R Das

2. Kakinada session (1923) Maulana Mohammad Ali

3. Madras session (1927) Dr. M.A. Ansari

4. Lucknow session (1916) Rajendra Prasad


How many pairs given above is/are correctly paired?

a) Only one pair


b) Only two pairs
c) Only three pairs
d) All the four above
39

Answer: (c)

● The Indian National Congress was founded at Bombay in December 1885.


● The early leadership – Dadabhai Naoroji, Pherozeshah Mehta, Badruddin Tyabji, W.C. Bonnerji,
Surendranath Banerji, Romesh Chandra Dutt, S. Subramania Iyer, among others – were largely from
Bombay and Calcutta.
● A retired British official, A.O. Hume also played a part in bringing Indians from the various regions
together.
● Formation of Indian National Congress was an effort in the direction of promoting the process of
nation building. In an effort to reach all regions, it was decided to rotate the Congress session among
different parts of the country.
● The President belonged to a region other than where the Congress session was being held.
Sessions of INC
● 1916: Lucknow.
○ President: A.C. Majumdar. So, pair 4 is not correctly matched.
○ Unity between two factions-Moderates and Extremists of Congress
○ Lucknow Pact signed between Congress and Muslim League to build political consensus
● 1922: Gaya.
○ President: C.R. Das. So, pair 1 is correctly matched.
○ CR Das and other leaders broke away from INC
○ Formation of Swaraj Party
● 1927: Madras.
○ President: Dr. M.A. Ansari. So, pair 3 is correctly matched.
○ Passed a resolution against the use of Indian troops in China, Iran and Mesopotamia.
○ Passed a resolution against boycott of Simon Commission
○ Adoption of resolution on Purna Swaraj
● 1923: Kakinada
○ President: Maulana Mohammad Ali, soon after this session he split from the Congress citing
the communal riots. So, pair 2 is correctly matched.
Therefore, option (c) is the correct answer.

Q.36) Which of the following was/were the objectives of the Swaraj party?
1. Attaining complete independence.
2. Establishing control over bureaucracy.
3. Control local and municipal bodies.
4. Establishing a federation of Asian countries to promote trade and commerce.
Select the correct answer using the codes given below:

a) 1 and 2 only
b) 2 and 3 only
c) 1, 2 and 3 only
d) 2, 3 and 4 only

Answer: (d)

Swaraj party:
● The Swaraj Party, established as the Congress-Khilafat Swaraj Party, was a political party formed in
India on 1 January 1923 after the Gaya annual conference in December 1922 of the National
Congress.
Formation
● The suspension of the non-cooperation movement in 1922 was met with an impressive measure
of logical inconsistencies among pioneers of the Congress Party.
○ While some wanted to continue non-cooperation, others wanted to end the legislature
boycott and contest elections.
○ The former were called no-changers, and later were called pro-changers
● In 1922, in the Gaya session of the Congress, C R Das (who was presiding over the session) moved a
proposal to enter the legislatures but it was defeated
40

○ Das and other leaders formed the Congress-Khilafat Swarajya Party with Das as the
president and Nehru as one of the secretaries
Objectives of Swaraj party:
● The Congress-Khilafat Swarajya Party or the Swaraj Party aimed for:
○ Speedy attainment of dominion status. So, statement 1 is not correct.
○ Obtaining the right to frame a constitution adopting such machinery and system as are
most suited to the conditions of the country and genius of the peoples
○ Establishing control over the bureaucracy. So, statement 2 is correct.
○ Obtaining full provincial autonomy
○ Getting people the right to control the existing machinery and system of government
○ Organising industrial and agricultural labour
○ Controlling the local and municipal bodies. So, statement 3 is correct.
○ Having an agency for propaganda outside the country.
○ Establishing a federation of Asian countries to promote trade and commerce. So,
statement 4 is correct.
○ Engaging in the constructive programmes of the Congress
Therefore, option (d) is the correct answer.

Q.37) With respect to August Offer, consider the following statements:


1. It was offered to secure participation of Indians in World War-II.
2. It proposed the viceroy's executive council to consist of Indian members only.
3. It proposed providing dominion status as an objective for India.
4. It proposed a representative Indian body to be formed after the war to draft the new constitution.
Which of the above statements is/are correct?

a) 1 and 3 only
b) 1, 3 and 4 only
c) 2, 3 and 4 only
d) 1, 2, 3 and 4

Answer: (b)
Explanation:
August Offer:
● The August Offer was made by Lord Linlithgow, Viceroy of India, in August 1940.
● The offer was made to secure support of Indians in British war efforts. So, statement 1 is correct.
Proposal
● The British government proposed dominion status as an objective for India. So, statement 3 is
correct.
● Following the war, a representative Indian body would be formed to draft a constitution for the
country. So, statement 4 is correct.
● The Viceroy's Council would be extended, allowing more Indians (not all) to participate than the
Britishers. So, statement 2 is not correct.
● An advising war council would be constituted after the war.
○ The British government, however, refused to grant complete independence.
● The British will retain control of the defense, finance, and home affairs, as well as all Republic of
India services.
● Viceroy further indicated that the disagreement between the Indian National Congress (INC) and
the Muslim League will be addressed before any constitutional reform.
● Minorities were guaranteed that their views will be taken into account in a future constitution.
Therefore, option (b) is the correct answer.
41

Q.38) With reference to the ‘Official Secret Act, 1923’, consider the following statements:
1. A person can be prosecuted under the act even if the act is done unintentionally.
2. The punishment for violating this act ranged from three to life imprisonment.
Which of the statements given above is/are correct?

a) 1 only
b) 2 only
c) Both 1 and 2
d) Neither 1 nor 2

Answer: (c)

Official Secrets Act, 1923:


● The Official Secrets Act of 1923 is India’s anti-espionage act.
● It states that actions which involve helping an enemy state against India are strongly condemned.
● It also states that one cannot approach, inspect, or even pass over a prohibited government site
or area.
● As per the act, helping an enemy state can be in the form of communicating a sketch, plan, a model of
an official secret, or of official codes or passwords, to the enemy.
● Prosecution and Penalties:
○ Punishments under the Act range from three to life imprisonment (if the intent is to
declare war against India – section 5 ). So, statement 2 is correct.
○ A person prosecuted under this Act can be charged with the crime even if the action was
unintentional and not intended to endanger the security of the state. So, statement 1 is
correct.
○ The Act only empowers persons in positions of authority to handle official secrets, and others
who handle it in prohibited areas or outside them are liable for punishment.
Therefore, option (c) is the correct answer.

Q.39) With reference to the Ambedkar circuit, consider the following places:
1. Mhow
2. Nagpur
3. Delhi
4. Mumbai.
Which of the places given above are part of the Ambedkar Circuit?

a) 1 and 2 only
b) 2 and 3 only
c) 1, 3 and 4
d) 1, 2, 3 and 4

Answer: (d)

● The Central government announced a special tourist circuit named Ambedkar circuit, which
encompasses five key sites associated with Dr B.R. Ambedkar.

Ambedkar Circuit:
● The government had first proposed the Ambedkar Circuit, or Panchteerth, in 2016, but the
conceptualisation of the plan could be held recently.
● The five cities in the tourist circuit as announced by the government are:0
○ Janma Bhoomi- Ambedkar’s birthplace in Madhya Pradesh’s Mhow. So, statement 1 is
correct.
○ Shiksha Bhoomi- the place in London where he stayed while studying in the UK.
○ Deeksha Bhoomi- the place in Nagpur where he embraced Buddhism. So, statement 2 is
correct.
○ Mahaparinirvan Bhoomi- the place of his demise in Delhi. So, statement 3 is correct.
○ Chaitya Bhoomi- the place of his cremation is in Mumbai. So, statement 4 is correct.
○ It connects only five important places related with the life of Ambedkar.
42

● Significance:
○ Focus on Tourism:
■ The idea is to attract tourists beyond the Dalit community, who mostly visit these
places as a pilgrimage.
■ The journey will include meals, ground transportation, and entry to the site.
○ Development of the Area:
■ The creation of special circuits allows the government to focus better on the
comprehensive development of all sites related to the Ambedkar theme,
including infrastructure, road and rail connectivity, and visitor facilities.
Therefore, option (d) is the correct answer.

Q.40) With respect to Savitri Bai Phule, consider the following statements:
1. She advocated inter-caste marriages, widow remarriage, and eradication of child marriage.
2. Savitribai became involved in relief work during the 1896 famine in Maharashtra
3. She started the Mahila Seva Mandal to raise awareness about women’s rights
Which of the statements given above is/are correct?

a) 1 and 3 only
b) 2 and 3 only
c) 1 and 2 only
d) 1, 2 and 3

Answer: (d)
Explanation:

● Savitribai Phule also advocated inter-caste marriages, widow remarriage, and eradication of child
marriage, sati, and dowry systems, among other social issues. So, statement 1 is correct.
● Savitribai became involved in relief work during the 1896 famine in Maharashtra and the 1897
Bubonic plague. So, statement 2 is correct.
● In 1852, Savitribai started the Mahila Seva Mandal to
raise awareness about women’s rights. Savitribai called for
a women’s gathering where members from all castes
were welcome and everybody was expected to sit on the
same mattress. She simultaneously campaigned against
child marriage, while supporting widow remarriage. So,
statement 3 is correct.

● More about Savitribai Phule:


○ She was a woman from the Mali community and
went on to become an educator, a challenger to
caste hierarchies and barriers, and a writer.
○ Born in Naigaon village of Maharashtra’s Satara district on January 3, 1831, to Khandoji
Nevse and Lakshmi.
○ Married off at an early age, her husband Jyotirao Phule is said to have educated her at
home.
○ In 1873, the Phule's set up the Satyashodhak Samaj (‘Truth-seekers’ society’), a platform
open to all, irrespective of their caste, religion or class hierarchies, with the sole aim of bringing
social equity.
○ As an extension, they started ‘Satyashodhak Marriage’ – a rejection of Brahmanical rituals
where the marrying couple takes a pledge to promote education and equality.
○ Savitribai became involved in relief work during the 1896 famine in Maharashtra and the
1897 Bubonic plague.
Therefore, option (d) is the correct answer.
43

Q.41) With reference to Gandhi’s initial struggle in India, consider the following statements:
1. Champaran Satyagraha was the first civil disobedience movement in response to high taxation
demanded by the Colonial planters in spite of crop failure.
2. Kheda Satyagraha occurred in response to tax hike despite Floods and famines struck the Kheda
region.
Which of the statements given above is/are correct?

a) 1 only
b) 2 only
c) Both 1 and 2
d) Neither 1 nor 2

Answer: (d)

Champaran Satyagraha
● Champaran Satyagraha of 1917 was the first civil disobedience movement of India.
● The Colonial planters had forced the peasants to grow indigo on 3/20 of the total land area (called
tinkathia system). So, statement 1 is not correct.
● When German synthetic dyes replaced indigo at the end of the nineteenth century, European planters
demanded high rents and illegal dues from the peasants in order to maximize their profits before the
peasants could switch to other crops.
● Furthermore, the peasants were forced to sell their produce at European-determined prices.
● Even though they were in the grip of a devastating famine, the British government imposed a heavy
tax on them and insisted on raising the rate.
● Gandhi was asked by Rajkumar Shukla, a local, to investigate the problems of farmers in the context of
indigo planters in Champaran, Bihar.
● When Gandhi arrived in Champaran, accompanied by Rajendra Prasad, Mazharul- Haq, Mahadev
Desai, Narhari Parekh, and J.B. Kripalani, the authorities ordered him to leave immediately.
● Gandhi defied the order and chose to face the consequences. This method of passive resistance or
civil disobedience in the face of an unjust order was novel at the time.
● Finally, the authorities relented and allowed Gandhi to conduct an investigation.
● Gandhi was able to persuade the authorities to abolish the tinkathia system and compensate the
peasants for the illegal dues extracted from them.
● As a compromise with the planters, he agreed to compensate them for only 25% of the money taken.
● Within a decade, the planters had abandoned the area. Gandhi had won India's first battle of civil
disobedience.
● Brajkishore Prasad, Anugrah Narayan Sinha, Ramnavmi Prasad, and Shambhusharan Varma
were also prominent leaders associated with the Champaran Satyagraha.

Kheda satyagraha
● Kheda Satyagraha of 1918 is known to be the first non-cooperation movement led by Mahatma
Gandhi.
● This satyagraha was centered on the peasant-Patidar community of Kheda, who refused to agree to
a 23 percent tax hike imposed on them despite a disastrous crop failure and outbreak of plague and
cholera. So, statement 2 is not correct.
● With the help of stalwarts such as Sardar Vallabhbhai Patel, Gandhiji's team members such as Indulal
Yagnik, Shankarlal Banker, and Mahadev Desai, among others, traveled across the countryside
raising awareness about farmers' rights.
● The revolt was notable for its adherence to discipline and unity.
● Even when the government seized the farmers' personal property, land, and livelihood for non-payment
of taxes, the vast majority of Kheda's farmers did not desert Sardar Patel.
● Gujaratis in other parts of the state who sympathized with the cause of the revolt assisted by sheltering
the protesting peasants' relatives and property.
● Those Indians who attempted to purchase the confiscated lands were socially shunned.
● Finally, the government attempted to reach an agreement with the farmers. It agreed to suspend the
tax for the current year and the following year, to reduce the rate increase, and to return all
confiscated property.
44

Ahmedabad Mill Strike 1918:

For the first time, Mahatma Gandhi led a satyagraha and a hunger strike for an industrial dispute between
the owners and the workers of a cotton mill in Ahmedabad.

● To compensate for wartime inflation, workers wanted a 50% raise.


● The employees went on a hunger strike as the mill owners were ready to pay only a 20% increase in
wages.
● For help in their struggle for justice, the mill workers went to Anusuya Sarabhai.
● Gandhi resolved the dispute between cotton mill owners and workers in Ahmedabad.

Bardoli Satyagraha:

● Floods and famines struck the Bardoli Taluka in early Gujarat in 1925, severely affecting crops.
Farmers' finances were affected as a result of this. The Bombay Presidency boosted tax rates by
22% despite the farmers' difficulties.
● Vallabhai Patel led his non-violent army at Bardoli with distinction. He split the taluk into camps
and enlisted the help of hundreds of men and women.
● Volunteers came from Hindu, Muslim, and Parsi communities, among others. Volunteers distributed
news bulletins, campaigns, and speeches from the taluk camps, instructing the groups on the
importance of being disciplined and prepared for adversity.
● The British Government was concerned that things might get out of hand, so the government
established the Maxwell-Broomfield commission to investigate the matter.
● The peasants were returned to the land that had been taken from them.
● The revenue was cut to 6.03 per cent.

Therefore, option (d) is the correct answer.

Q.42) Hindu-German conspiracy which was a significant event by the Indian nationalists took place during which
of the following events:

a) Civil Disobedience Movement


b) After suspension of Non-Cooperation Movement
c) World War-I
d) During communal riots of 1945-46.

Answer: (c)

Hindu-German Conspiracy:
● The Hindu German conspiracy or the German Plot was a series of plans by Indian nationalist
groups to attempt Pan-Indian rebellion against the British Raj during World War I, formulated
between the underground revolutionaries and exiled or self-exiled nationalists in America and various
European countries. So, option (c) is correct.
● The Ghadar party in the US and The Indian Independence committee in Germany were part of it.⁣
● Germans supported the conspiracy financially and militarily, so it was named after it.
● The original plan consists of 3 major parts- ⁣
○ Simultaneous revolution in other British colonies like Burma and Singapore along with
India.⁣
○ Multiple revolt activities in Northern India under Gadar Party, which later came to be known
as The Ghadar conspiracy.
■ Similar activities to be hosted in Eastern India all across Bihar, Bengal and Odisha
under a secret underground society Jugantar.
■ The last series of attacks would be taken up by Indian soldiers of the British army from
within under leadership of Jugantar’s Commander in chief Bagha Jatin. ⁣
○ German navy ships' direct attack over Andaman Islands and Madras, which will be
supported by Indian freedom fighters jailed in prisons. ⁣
45

● All this was to be undertaken simultaneously, coordinated by the Indian Independence Committee
formed in Berlin. The conspiracy was planned to trigger a national mutiny on lines of 1857 mutiny.⁣
Therefore, option (c) is the correct answer.

Q.43) With reference to the Mangarh massacre, consider the following statements:
1. The massacre occurred on the border of Gujarat and Rajasthan.
2. The Kol tribes agitated against the British policies.
3. The movement was initiated by Guru Govindgiri who raised a front against local rulers
Which of the statements given above is/are correct?

a) 1 and 2 only
b) 2 and 3 only
c) 1 and 3 only
d) 1, 2 and 3

Answer: (c)

The Mangarh massacre


● Six years before the Jallianwala Bagh massacre in 1919 there was another brutal repression and
massacre.
● The massacre took place on the border of Rajasthan and Gujarat. So, statement 1 is correct.
● Over 1,000 people, all members of the Bhil tribe, lost their lives.
● Mangarh, in the Aravali hills was washed in blood on 17th November 1913.
● Bhils, a tribal community living across Maharashtra, Gujarat, Rajasthan and Madhya Pradesh faced
great harassment under the old feudal structure and British rule made it worse.
● By the early 20th century, the Bhils especially in Rajasthan and Gujarat, mostly worked as bonded
labor. The Bhils agitated against the British policies. So, statement 2 is not correct.
● The great famine of 1899-1900 across the Deccan and the Bombay Presidency claimed over 6 lakh
victims and the tribals were among the worst affected.
● The princely states such as Banswara and Santrampur were particularly affected by the drought.
● From this tragedy emerged a social reform movement that aimed at bettering the lot of the
marginalized.
● The movement was led by social reformer Govind Giri, who was also popularly called as Govind
Guru. So, statement 3 is correct.
● To address the challenges faced by the community, Govind Giri started the Bhagat Movement in
1908.
● This was a movement to propagate orthodox Hindu practices like vegetarianism and abstaining from
alcohol among the Bhils.
● He also encouraged them to reject bonded labor and fight for their rights.
● The princely states of Dungarpur, Banswara and Santrampur where this movement was strongest were
quite vary of this Bhil 'awakening'.
● The growing awareness among the local Bhils meant that they demanded better wages from the rulers
and the British.
● As they took up arms and stopped work, the local economy suffered.
● In October 1913, Govind Giri and his followers reached Mangarh, which was situated in the middle
of dense forests, on the border of Banswara and Santrampur state.
● They were planning to revolt against the princely states of Banswara and Santrampur, and
establish a Bhil state. The worried rulers turned to the British for help.
● The British political representative to the region, R.E Hamilton, decided to take action.
● The combined forces of the princely states of Banswara, Dungarpur and Santrampur, along with the
Bhil Corps of Mewar state (Udaipur) were asked to surround the hill.
● An ultimatum was given to the gathered Bhils to disperse by 15th November 1913.
● When the Bhils refused to surrender and disperse, the gathering was literally bombarded with
bullets and artillery fire from all sides.
● After the massacre, hundreds of Bhil protesters including Govind Giri were arrested.
Therefore, option (c) is the correct answer.
46

Q.44) With reference to the Cornwallis code, 1793, consider the following statements:
1. Zamindars were the sole owner of land and had the rights to transfer land.
2. It halted private trade carried on by Company servants in the names of their family and decreased the
public servant salaries.
Which of the statements given above is/are correct?

a) 1 only
b) 2 only
c) Both 1 and 2
d) Neither 1 nor 2

Answer: (a)

● After the American Revolution, Cornwallis was appointed as the Governor-General of India in
1786. He introduced administrative and judicial reforms, known as the Cornwallis Code, aimed at
improving governance and legal systems in Bengal.
● The Cornwallis code recognised zamindars (landlord) as the sole owner of land and gave full
rights to transfer or donate such land. The government revenue was permanently fixed and if the
zamindar failed to pay it, the land would be auctioned off to realize the government revenues. So,
statement 1 is correct.
● Until 1793 (before the introduction of Cornwallis code) a British citizen could only be tried by the
Supreme Court of Calcutta, and no Indians could file a case against any employee of the East India
Company. This discrimination was removed by this code, and everyone declared equal in the eyes
of law. The natives (Indians) were now entitled to file a case against government officials including
Europeans.
● Cornwallis outlawed the receipt of bribes and gifts, halted private trade carried on by Company
servants in the names of their family, raised public servant salaries, and promoted only Englishmen to
positions of responsibility.
● Cornwallis decreased the number of districts from 25 to 23, dissolved numerous jobs, stripped
collectors of their civil justice functions, and made selections based solely on merit. So,
statement 2 is not correct.
● He delegated trade-related responsibilities to the trade-board, reduced the number of board
members, and selected commission-based representatives.
● For the purpose of ensuring peace and order, Cornwallis erected police posts and appointed police
sub-inspectors.
● Lord Cornwallis removed the executive from the judiciary, erased the division between civil and
revenue courts, and formed a civil court system, with courts of Munsifs, Registrars, district
courts, four province courts, and the Sardar Diwani Adalat at the top.
● Circuit courts in districts and four provincial courts in Patna, Dacca, Murshidabad, and Calcutta
were established for criminal trials, with Sardar Nizamat Adalat at the head.
● Lord Cornwallis is known for his military leadership, his contributions to British colonial
administration, and his role in pivotal moments of history, including the American Revolutionary
War and British India. His legacy is complex, as he was both praised for his military skills and criticized
for his policies and actions during these turbulent times.
Therefore, option (a) is the correct answer.

Q.45) With reference to the Khilafat Movement, consider the following statements:
1. Shaukat Ali and Muhammad Ali were the leaders of the Khilafat agitation.
2. The purpose of the movement was to pressure the British government to preserve the authority of
Khalifa over the Muslim sacred places in the Ottoman empire.
3. The Muslim League did not support the movement.
Which of the statements given above are correct?

a) 1 and 2 only
b) 2 and 3 only
c) 1 and 3 only
d) 1, 2 and 3
47

Answer: (a)

● In 1920 the British imposed a harsh treaty on the Turkish Sultan or Khalifa. People were furious
about this as they had been about the Jallianwala massacre. Also, Indian Muslims were keen that the
Khalifa be allowed to retain control over Muslim sacred places in the erstwhile Ottoman Empire.
● The Indian Muslims were incensed when they discovered that their loyalty had been purchased
during the War by assurances of generous treatment of Turkey after the War - a promise British
statesmen had no intention of fulfilling.
● The Muslims regarded the Caliph of Turkey as their spiritual head and were naturally upset when
they found that he would retain no control over the holy places in the erstwhile Ottoman Empire, the
jazirat-ul-Arab (Arabia, Syria, Iraq and Palestine). The leaders of the Khilafat agitation, Mohammad Ali
and Shaukat Ali, now wished to initiate a full-fledged movement. So, statements 1 and 2 are
correct.
● Although Mahatma Gandhi was in favour of launching Satyagraha and non-cooperation against the
Government on the Khilafat issue, the Congress was not united on this form of political action.
○ The Congress, later, felt inclined to provide its support as it was a golden opportunity to
unite the Hindus and Muslims and to bring Muslim participation in such mass movements.
○ The Muslim League also decided to give full support to the Congress and its agitation on
political questions. So, statement 3 is not correct.
● Gandhiji, who had been in close touch with the Khilafat leaders for quite some time, and was a special
invitee to the Khilafat Conference in November 1919, had all along been very sympathetic to their
cause, especially because he felt the British had committed a breach of faith by making promises that
they had no intention of keeping.
● In February 1920, he suggested to the Khilafat Committee that it adopt a programme of non-violent
non-cooperation to protest the Government’s behaviour. On 9 June 1920, the Khilafat Committee at
Allahabad unanimously accepted the suggestion of non-cooperation and asked Gandhiji to lead the
movement. The movement was launched formally on 1 August 1920.
Therefore, option (a) is the correct answer.

Q.46) With reference to Trade organizations, consider the following pairs:


S. No. Trade organisation Created by

1. National Social Service League N. M. Lokhande

2. Bombay Mill-hands Association N. M. Joshi

3. Working Men’s club Sasipada Banerjee

4. Workers Welfare Society N. A. Talcherkar


How many pairs given above is/are correctly matched?

a) Only one pair


b) Only two pairs
c) Only three pairs
d) All the four above

Answer: (b)

Pre phase-1918: Initial Stage of Labour Movement


● In March 1862, the first-ever strike of Indian Industrial workers took place when 1200 Railway
workers of Howrah Station demanded an eight-hour workday.
● In 1870, the first labour organization Working Men's Club was founded in Calcutta by Sasipada
Banerjee. He also published in the journal 'Bharat Sramajibi'. So, pair 3 is correctly matched.
● In 1875, Sorabjee Sharpoorji Bengalee of Mumbai organized the labour agitation in Bombay against
the appalling conditions of workers, especially women and children. Thus, the first organized labour
48

agitation was organised by S. S. Bengalee. It resulted in the formation of the first factory
commission in the country in 1875.
● In 1875, Narayan Meghaji Lokhande was the first to represent the grievances of the Indian working
class before the Labour Commission of Bombay. From 1880 onwards, N. M. Lokhande also started
the publication of Deenbandhu newspaper in Bombay.
● In 1877, the first Industrial strike took place in the Nagpur Empress Mill,
● demanding a wage hike.
● In 1878, S. S. Bengalee drafted a bill and tried to pass it in the Bombay legislative council to provide
better working conditions to the labourers. In 1881, the first factory Act of 1881 was passed by the
British government in India.
● In 1890, N. M Lokhande set up the first association of Indian workers, the 'Bombay Mill-Hands
Association' in Bombay. That's why N. M. Lokhande was also known as the founding father of the
Trade Union Movement in India. So, pair 2 is not correctly matched.
● In 1897, the Amalgamated Society of Railway Servants of India was established. The first strike by
the Great Indian Peninsula railways took place in 1899, which was also supported by Tilak's
newspaper, Kesari and Mahratta.
Swadeshi Upsurge
● During the Swadeshi upsurge, the working classes in India came with wider political issues. Mass
level strikes were organized by Apurba Kumar Ghosh, Ashwini Coomar Banerjea, Premtosh Bose,
and Prabhat Kumar Roy Chaudhuri, mainly in government press, jute industry, and railways.
● Following this, worker organizations were formed across the country. Important workers'
organizations include Calcutta's Printer Union (1905), Bombay Postal Union (1907), and Kamgar
Hitwardhak Sabha (1910). All of these labour unions aimed to promote the welfare of workers and
spread literacy among them.
Kamgar Hitwardhak Sabha
● Kamgar Hitwardhak Sabha or the Workers Welfare Society was formed in 1910 by N. A. Talcherkar,
S K Bole, B R Nare, S W Patil and others. The constituted the mill workers as well as other
employees and representatives from general public and occupations like law and medicine. It
supported the reduction of working hours to 12 per day and urged the claims of workers for
industrial compensation and education. So, pair 4 is correctly matched.
● The objectives of the Sabha were:
○ To give relief to workmen in indigent circumstances;
○ To promote education among them by means of night classes and instructive lectures;
○ To wean all workers from harmful customs;
○ To find them help in times of stress;
○ To settle amicably disputes between employers and employed;
○ To render work-people medical and legal help when necessary; and
○ To make all efforts to ameliorate the condition of the working classes generally.
● In 1908, the biggest strike took place when Bal Gangadhar Tilak was arrested on charges of
sedition. It was a week-long protest held by Mumbai Mill workers.
● In 1911, the National Social Service league in Bombay was established by N. M. Joshi (the greatest
exponent of the labour movement in India). So, pair 1 is not correctly matched.
Therefore, option (b) is the correct answer.

Q.47) With reference to Munda Rebellion, consider the following statements:


1. It was a revivalist movement to restore the primitive character of the Munda society.
2. The Munda were a tribe based in Madhya Pradesh.
3. Chotanagpur Tenancy Act was an outcome of the Munda rebellion.
Which of the statements given above is/are correct?

a) 1 only
b) 2 and 3 only
c) 1 and 3 only
d) 1, 2 and 3

Answer: (c)
49

Munda rebellion
● The Munda Ulgulan (rebellion) is one of the most prominent tribal revolts in the history of Indian
Independence.
● Even though the end was not favourable, it sent a message across the borders that the tribal people
know how to raise their voice and to what extent.
● The Munda were a tribe based in Chhota Nagpur of Jharkhand whose means of living was
agriculture. So, statement 2 is not correct.
● The cause of this uprising was similar to that of other rebellions – the British Colonizers, Zamindars
and Missionaries.
The Rebellion

● Birsa Munda spearheaded the tribal movement.


● He strived for a positive political programme, his object being the attainment of independence, both
religious and political.
● The movement sought the assertion of the rights of the Mundas, as the real proprietors of the soil.
● He called upon the Mundas to fight against superstition, give up animal sacrifice, stop taking
intoxicants, to wear the sacred thread and retain the tribal tradition of worship in the sarna or the sacred
grove.
● The rebellion was essentially a revivalist movement, which sought to purge Munda society of all
foreign elements and restore its pristine character. So, statement 1 is correct.
● Further, by the 1890s, he was mobilizing people and enraging the tribal people in the region.
● In 1894, he declared a revolt against the British and the dikus and declared to create a ‘Munda Raj’.
● Under his leadership, the villagers attacked the police stations, churches and government properties in
1899.
● However, on 9 January, 1900, the rebels were defeated. Birsa was captured and died in jail. Nearly
350 Mundas were put on trial and of them three were hanged and 44 transported for life.
Significance of the Movement
● Although the rebellion could not reach the desired end, it left a significant impact on the tribal movement
of India.
● It showed that the tribal people had the capacity to protest against injustice and express their anger
against colonial rule.
● The British enacted the Chotanagpur Tenancy Act, 1908 which restricted the transfer of tribal land to
non-tribal people. So, statement 3 is correct.
● The “Khuntkatti” rights were recognised and ”Beth begari” was banned.
● Consequently, tribals won a degree of legal protection for their land rights.
Therefore, option (c) is the correct answer.
50

Q.48) Which of the following statements is correct with respect to the Alipore conspiracy case?

a) Madan lal Dhingra assassinated Curzon Wyllie in 1909.


b) A bomb was thrown at the carriage of Lord Hardinge.
c) Chapekar brother assassinated Plague commissioner W C Rand
d) Khudiram Bose and Prafull Chaki threw bombs on the carriage of Kingsford.

Answer: (d)

Alipore Conspiracy Case:


● Additionally called the Muraripukur scheme or Manicktolla bomb intrigue.
● Douglas Kingsford was a disliked British Chief Magistrate who was the objective of the bomb tossed
at Muzaffarpur (Northern Bihar).
● Sadly, the carriage on which the bomb was focused contained two English women and not
Kingsford. The two ladies passed on in the assault.
● Progressives who tossed the bomb were Prafulla Chaki and Khudiram Bose.
● Chaki ended his life while Bose, at that point just 18 years old, was contracted and condemned to the
terrible bug by hanging. So, option (d) is correct.

Madan lal Dhingra assassinated Curzon Wyllie

● Madan Lal Dhingra was a great revolutionary from Punjab, associated with the Indian Home Rule
Society, The Abhinav Bharat Society and the Indian House in London.
● On July 1, 1909 he shot dead Curzon Wyllie, an adviser to the secretary of state of India, and Cowas
Lolcaca at the meeting of the Indian National Association in London to avenge the atrocities committed
by the British in India. So, option (a) is not correct.

Delhi Conspiracy case

● The Delhi Conspiracy case, also known as the Delhi-Lahore Conspiracy, refers to an attempt
made in 1912 to assassinate the then Viceroy of India, Lord Hardinge by throwing a local self-made
bomb, on the occasion of transferring the capital of British India from Calcutta to New Delhi.
● Hatched by the Indian revolutionaries underground in Bengal and Punjab and headed by Rash Behari
Bose, the conspiracy culminated in the attempted assassination on 23 December 1912, when a
homemade bomb was thrown into the Viceroy's howdah as the ceremonial procession was moving
through the Chandni Chowk suburb of Delhi. So, option (b) is not correct.

Chapekar brothers case

● On 22 June 1897, brothers Damodar Hari Chapekar and Balkrishna Hari Chapekar assassinated
British official W. C. Rand and his military escort Lieutenant Ayerst at Pune, Maharashtra. This was
the first case of militant nationalism in India after the 1857 Revolt.
● During 1896-97, there was a bubonic plague in Pune (Poona). It is also known as Poona Plague.
The government had set up a special plague committee in the year 1897 to deal with the menace of
plague and control the spread of the disease. Charles Walter Rand (W.C. Rand) was the chairman of
this committee.
● Instead of appointing doctors, the Plague Commission had deployed more than 800 officers and
soldiers in Pune.
● People were not being allowed to perform the last rites of their disease-affected relatives. The
persecution of the local people by the British soldiers had started increasing.
● The continued persecution of the Rand Commission prompted the Chapekar brothers and other
members of the revolutionary Chapekar Club to take action against Rand.
● The Chapekar brothers had formed a revolutionary organization called “Chapekar Club” for physical
and military training. So, option (c) is not correct.

Therefore, option (d) is the correct answer.


51

Q.49) With reference to Mulshi Peta Satyagraha, consider the following statements:

1. The Satyagraha was started against a dam being built on the river Mula in Maharashtra.
2. The main leaders of the struggle were Bhuskute and Senapati Bapat.
3. Mahatma Gandhi and the Mulshi Peta Satyagrahis were imprisoned in the Yerwada Jail.
4. This Satyagraha is mentioned in a book by Rajendra Vohra.
Which of the statements given above is/are correct?

a) 1 and 2 only
b) 2, 3 and 4 only
c) 1, 3 and 4 only
d) 1, 2, 3 and 4

Answer: (d)

● The Mulshi Peta Satyagraha against a dam being built by the Tata company on the river Mula in
Maharashtra is one of the first anti-dam struggles of India, fought nearly a hundred years ago.
However, not much is known about this important and powerful struggle, though it has been well
documented in the book ‘Mulshi Satyagraha’, by Rajendra Vohra. So, statements 1 and 4 are
correct.
● The main leaders of the struggle were Bhuskute and Senapati Bapat and it had a very large
participation of women too. So, statement 2 is correct.
● Mulshi Peta satyagrahis in Yerwada Prison were flogged for refusing to work. In June 1923,
Gandhiji sought Jail Superintendent’s permission to meet Mulshi Peta prisoners.
● Gandhiji was Arrested near Sabarmati Ashram for writing three articles in Young India. Sentenced to six
years imprisonment in the Yerwada Prison. Released from Yervada prison on 5 February, 1924
unconditionally after an operation on 12 January, 1924. Statement 3 is correct.
Therefore, option (d) is the correct answer.

Q.50) With reference to Important books written during the Indian freedom struggle, consider the following pairs:

S. No. Name of the Book Author

1. Ghulam Giri Tarabai Shinde

2. Economic History of India Dadabhai Naoroji

3. Precepts of Jesus Swami Vivekanand

4. Unhappy India Lala Lajpat Rai


How many pairs given above is/are correctly paired?

a) Only one pair


b) Only two pairs
c) Only three pairs
d) All the four above

Answer: (a)

● Important books were written during the Indian freedom struggle

Name of the Book Author

Ghulam Giri Jyotiba Phule. So, pair 1 is not correctly matched.


52

Causes of the Indian Mutiny Sir Syyed Ahmed Khan

To all fighters of freedom, Why Socialism J.P. Narayan

Pakhtoon Khan Abdul Ghaffar Khan

Problems of the East Lord Curzon

My Indian Years Lord Hardinge II

Economic History of India R.C. Dutt. So, pair 2 is not correctly matched.

Precepts of Jesus Raja Ram Mohan Roy. So, pair 3 is not correctly
matched.

A Gift of Monotheists Raja Ram Mohan Roy

Satyarth Prakash Swami Dayanand Saraswati

The Indian Struggle S.C. Bose

Unhappy India Lala Lajpat Rai. So, pair 4 is correctly matched.

The Indian War of Independence V. D. Savarkar

India Divided Rajendra Prasad


Therefore, option (a) is the correct answer.

Q.51) He was an Indian philosopher, social leader and founder of the Arya Samaj. His vision of India included a
classless and casteless society, a united India and an India free from foreign rule. He took inspiration from the
Vedas and considered them to be ‘India’s Rock of Ages’, the infallible and the true original seed of Hinduism. He
gave the slogan “Back to the Vedas”. Which of the following personality has been referred to in the above
passage?

a) Raj Rammohan Roy


b) Swami Dayanand Saraswati
c) Ishwarchandra Vidyasagar
d) Keshub Chandra Sen

Answer: (b)

Maharishi Dayanand Saraswati

● Swami Dayanand Saraswati was born on 12th February 1824 in Tankara, Gujarat in a Brahmin family.
● He was earlier named Mool Shankar Tiwari as he was born during Mool Nakshatra.
● Dayananda’s views were published in his famous work, Satyarth Prakash (The True Exposition).

Contribution to the Society:

● He was an Indian philosopher, social leader and founder of the Arya Samaj.
○ Arya Samaj is a reform movement of Vedic dharma and he was the first to give the call for
Swaraj as "India for Indian" in 1876.
○ The first Arya Samaj unit was formally set up by him at Mumbai (then Bombay) in 1875
and later the headquarters of the Samaj were established at Lahore.
● His vision of India included a classless and casteless society, a united India (religiously, socially and
nationally), and an India free from foreign rule, with Aryan religion being the common religion of all.
53

● He took inspiration from the Vedas and considered them to be ‘India’s Rock of Ages’, the infallible
and the true original seed of Hinduism. He gave the slogan “Back to the Vedas”.
● He subscribed to the Vedic notion of chaturvarna system in which a person was not born in any caste
but was identified as a brahmin, kshatriya, vaishya or shudra according to the occupation the person
followed. So, option (b) is correct.

Contribution to the Education System:

● He introduced a complete overhaul of the education system and is often considered as one of the
visionaries of modern India.
● The DAV (Dayanand Anglo Vedic) schools came into existence in 1886 to realize the vision of Swami
Dayanand Saraswati.
● The first DAV School was established at Lahore with Mahatma Hansraj as the headmaster.

Therefore, option (b) is the correct answer.

Q.52) With reference to Home Rule Leagues of Tilak and Besant, consider the following statements:

1. Tilak’s league was restricted to Maharashtra whereas Besant’s League covered the rest of India except
Bombay.
2. Anglo-Indians and non- brahmins from the south India did not join either of the home rule league
3. Besant’s League was less organised as compared to Tilak’s league.
Which of the statements given above is/are correct?

a) 1 and 3 only
b) 2 and 3 only
c) 3 only
d) 1 and 2 only

Answer: (b)

The Home Rule Movement


● The home rule movement was the Indian response to the First World War in a less charged but in a
more effective way.
● With people already feeling the burden of war time miseries caused by high taxation and a rise in prices,
Tilak and Annie Besant ready to assume the leadership the movement started with great vigour.
● Two Indian Home Rule Leagues were organised on the lines of the Irish Home Rule Leagues and
they represented the emergence of a new trend of aggressive politics.
● The League campaign aimed to convey to the common man the message of home rule as
self-government.
Objectives of Home Rule Movement:
● To achieve self-government in India.
● To promote political education and discussion to set up agitation for self-government.
● To build confidence among Indians to speak against the government’s suppression.
● To demand a larger political representation for Indians from the British government.
● To revive political activity in India while maintaining the principles of the Congress Party.
Home Rule Leagues of Tilak and Besant
● Tilak and Besant had set up their separate Home Rule Leagues.
● Tilak set up his Indian Home Rule League in April 1916. Annie Besant set up her All-India Home
Rule League in September 1916.
● Tilak’s league was restricted to Maharashtra (excluding Bombay city), Karnataka, Central
Provinces, and Berar. It had six branches. Annie Besant’s league was set up as All-India Home Rule
League and it covered the rest of India (including Bombay city). It’s headquarter was in Madras and
had 200 branches. So, statement 1 is not correct.
● Muslim League did not support in setting up both Besant’s as well as Tilak’s Home Rule League.
Also, Anglo-Indians, most of the Muslims and non- brahmins from south India did not join the home
54

rule. They believed that by home rule it aims to spread Hindu, high caste majority. So, statement 2 is
correct.
● Annie Besant’s League was loosely organised as compared to Tilak’s league. In her league- three
members could form a branch whereas in case of Tilak’s League each of the six branches had a clearly
defined area and activities. Also, there was no organized method for passing the instructions in
Besant’s league. So, statement 3 is correct.
Therefore, option (b) is the correct answer.

Q.53) Mahatma Gandhi undertook fast unto death in 1932, mainly because

a) Round Table Conference failed to satisfy Indian political aspirations


b) Congress and Muslim League had differences of opinion
c) Ramsay Macdonald announced the Communal Award
d) To stop Civil Disobedience Movement as it turned violent

Answer: (c)

Communal Award

● Communal Award (MacDonald Award) means a group of elections for depressed classes and
minorities.
● On August 16, 1932, British Prime Minister Ramsay MacDonald established the Communal Award.
● It was introduced following the Round Table Conference and expanded the separate electorate to
lower Classes and other minorities. It was also known as the MacDonald Award.
● The Indian Councils Act 1909 established a distinct electorate for Muslims, which was later
expanded to include Sikhs, Indian Christians, Anglo-Indians, and Europeans by the Government of
India Act 1919.
● Gandhiji reacted strongly to the proposal of granting the right of a separate electorate to the
depressed classes. He regarded the Depressed Classes as an integral part of Hindu society.
● He had pinned his hopes for the welfare on the firm belief that the Hindus would do full social justice to
that section of society whom they had exploited for centuries and would fully integrate them within their
fold.
● To persuade the recalcitrant Ambedkar to accept his viewpoint on this, Gandhi, then in the Yerwada
Jail, resorted to a fast unto death.
● In an effort to save his life, the Poona Pact was concluded on 25 September 1932.
● As promised, Gandhi ended his fast after the signing of the pact. He found in this victory his
political and social goals. So, option (c) is correct.

Therefore, option (c) is the correct answer.

Q.54) The Regulating act of 1773 was introduced by the British government for which of the following reason?

a) To establish provincial autonomy


b) To establish a central administrative system in British India.
c) To give economic as well as legislative powers to the East India Company
d) To curb the corruption prevailing among the officers of the East India Company.

Answer: (b)

Regulating Act of 1773

● The Regulating act of 1773 was introduced to establish a central administrative system in British
India. So, option (b) is correct.
● The act was brought to regulate the activities of the British East India company. It was an initiative
of the British parliament to bring an administrative reform in British India.
55

Key Provisions of the Act (Major features):

● Introduction of the office of the Governor-General of Bengal: The office of the Governor of Bengal
was redesignated as the Office of the Governor of the Presidency of Fort William, also known as
Governor-General of Bengal. Lord Warren Hastings was the first person to hold this designation.
● Creation of Executive Council to Assist the Governor-General: The Executive Council of four
members was created to assist the Governor-General.
● Governors of Bombay and Madras presidencies subordinate to the Governor-general of Bengal:
The Governors of Bombay and Madras were made subordinate to the Governor General of Bengal,
thereby making the Governor General of Bengal as the ultimate authority.
● Establishment of the Supreme Court at Judicature at Fort William: In the year 1774, the Supreme
Court of Judicature at Fort William was established at Calcutta with one Chief Justice and three other
judges. The jurisdiction of this court extended to all areas lying under the Bombay, Madras and Bengal
Presidency.
● Reforms to curb corruption: This act brought prohibition on the servants of the company from
engaging in any private trade or accepting bribes and gifts from the local people. The directors of the
company were to be elected for a period of five years and one-fourth of them used to retire every year.
There was no procedure for re-election available.

Therefore, option (b) is the correct answer.

Q.55) With reference to the constitutional developments during the British rule, consider the following statements:

1. The Government of India Act, 1919 introduced dyarchy at the executive level of the provinces.
2. The Government of India Act, 1935 abolished the dyarchy from the provinces and introduced it at the
central level.
3. The Government of India Act, 1909 introduced bicameral legislature at the central level.

Which of the statements given above is/are correct?

a) 1 and 2 only
b) 2 and 3 only
c) 3 only
d) 1, 2 and 3

Answer: (a)

Government of India Act, 1919:


● The Government of India Act 1919 was an act of the British Parliament that sought to increase the
participation of Indians in the administration of their country.
● A bicameral legislature was set up at the Central level with two houses – Legislative Assembly
and the Council of State. So, statement 3 is not correct.
● Dyarchy was introduced at the executive level of the provincial government i.e. there were two
classes of administrators – Executive councilors and ministers. So, statement 1 is correct.
● The Governor was the executive head of the province. The subjects were divided into two lists –
reserved and transferred.
● The governor was in charge of the reserved list along with his executive councilors. The
subjects under this list were law and order, irrigation, finance, land revenue, etc.

Government of India Act, 1935


● The Government of India Act, 1935 was passed by the British Parliament in Aug 1935.
● This act ended the system of dyarchy introduced by Government of India Act 1919 and
provided for establishment of a Federation of India to be made up of provinces of British
India and some or all of the Princely states.
● The Act established an All-India Federation, which included both British Indian Provinces
and the Indian States. The Instrument of Accession specified the conditions under which a
state might join the federation.
56

● The Act of 1935 abolished dyarchy at the Provincial level and introduced it at the Centre.
So, statement 2 is correct.
● It further extended the principle of communal representation by providing separate
electorates for depressed classes (scheduled castes), women and labour (workers).
● The Act established a Federal Court with one Chief Justice and no more than eighty-six
judges.
● The Government of India Act 1935 abolished the Council of the Secretary of State for
India, which was created in 1858.
● It provided for the establishment of a Reserve Bank of India to control the currency and
credit of the country.
● It also provided for the establishment of not only a Federal Public Service Commission but
also a Provincial Public Service Commission and Joint Public Service Commission for
two or more provinces.
Government of India Act, 1909:
● The Morley-Minto reforms named after the Secretary of State for Indian Affairs Lord John Morley and
the Viceroy Lord Minto was the alternative name given to Indian Councils Act 1909.
● It introduced for the first time the method of election, an attempt to widen the scope of legislative
councils, placate the demands of moderates in Indian National Congress and to increase the
participation of Indians in the governance. The Act amended the Indian Councils Acts of 1861
and 1892.
Features of the 1909 Act
● It considerably increased the size of the legislative councils, both Central and provincial. The
number of members in the Central Legislative Council was raised from 16 to 60. The number of
members in the provincial legislative councils was not uniform.
● It retained official majority in the Central Legislative Council but allowed the provincial legislative
councils to have non-official majority.
● The elected members were to be indirectly elected. The local bodies were to elect an electoral
college, which in turn would elect members of provincial legislatures, who in turn would elect members
of the central legislature.
● It enlarged the deliberative functions of the legislative councils at both the levels. For example,
members were allowed to ask supplementary questions, move resolutions on the budget, and so on.
● It provided (for the first time) for the association of Indians with the executive Councils of the
Viceroy and Governors. Satyendra Prasad Sinha became the first Indian to join the Viceroy’s
Executive Council. He was appointed as the law member.
● It introduced a system of communal representation for Muslims by accepting the concept of
‘separate electorate’. Under this, the Muslim members were to be elected only by Muslim voters. Thus,
the Act ‘legalised communalism’ and Lord Minto came to be known as the Father of Communal
Electorate.
● It also provided for the separate representation of presidency corporations, chambers of
commerce, universities and zamindars.

Therefore, option (a) is the correct answer.

You might also like